[obm-l] Soma de cossenos

2021-02-15 Por tôpico heitor326
Prove que cos2pi/17+cos18pi/17+cos26pi/17+cos30pi/17=(17^(1/2)-1)/4

-- 
Esta mensagem foi verificada pelo sistema de antivírus e
 acredita-se estar livre de perigo.

=
Instruções para entrar na lista, sair da lista e usar a lista em
http://www.mat.puc-rio.br/~obmlistas/obm-l.html
=


[obm-l] Re: [obm-l] Soma de raízes quadradas

2020-04-05 Por tôpico Anderson Torres
Em seg., 17 de fev. de 2020 às 12:43, Vanderlei Nemitz
 escreveu:
>
> Boa tarde!
> Existe uma fórmula fechada para a soma das raízes quadradas dos n primeiros 
> números naturais?
>

1 - Duvido.

2 - Qual a necessidade prática disso?

> Muito obrigado!
>
> --
> Esta mensagem foi verificada pelo sistema de antivírus e
> acredita-se estar livre de perigo.

-- 
Esta mensagem foi verificada pelo sistema de antiv�rus e
 acredita-se estar livre de perigo.


=
Instru��es para entrar na lista, sair da lista e usar a lista em
http://www.mat.puc-rio.br/~obmlistas/obm-l.html
=


Re: [obm-l] Soma surpreendentemente inteira

2020-03-14 Por tôpico qedtexte

Sauda,c~oes, oi Pedro,

Colocando "sum 3/(cos((24pi n)/180)-1) n=1 to 7" no WolframAlpha
o resultado  -56.

Mas no sei como fazer. Eu tentaria fazer 1=cos0 e
cos(24n)-cos0=-2sin^2(12n)

Colocando no WA

sum 3/(-2sin^2((12pi n)/180)) n=1 to 7; sum 3/(-2sin^2((pi n)/15)) n=1 to 7

ele retorna alguns clculos e volta a encontrar -56.

Agora a soma fica-3/2 sum_(n=1)^7 1/(sin^2(( n)/15))

Usando a lgebra dos nmeros complexos pode ser que saia.

Lus
--
Esta mensagem foi verificada pelo sistema de antiv�rus e
acredita-se estar livre de perigo.



[obm-l] Soma surpreendentemente inteira

2020-03-11 Por tôpico Pedro Cardoso
Olá, amigos.

Gostaria de ajuda para calcular a segunte soma:

Soma com n variando de 1 a 7 de
3/(cos(24n)-1)

Com o argumento do cos em graus

Aparentemente essa soma é 56, não consegui entender porque

-- 
Esta mensagem foi verificada pelo sistema de antiv�rus e
 acredita-se estar livre de perigo.



Re: [obm-l] Re: [obm-l] Soma de raízes quadradas

2020-02-18 Por tôpico Maikel Andril Marcelino
http://www.mat.puc-rio.br/~nicolau/olimp/obm-l.html


Atenciosamente,

Maikel Andril Marcelino
Assistente de Aluno
Coordenadoria de Apoio Acadêmico - COAPAC/IFRN-SPP
Instituto Federal do Rio Grande do Norte
Campus São Paulo do Potengi

(84) 9-9149-8991 (Contato)
(84) 8851-3451 (WhatsApp)

De: owner-ob...@mat.puc-rio.br  em nome de Hermann 

Enviado: terça-feira, 18 de fevereiro de 2020 11:07
Para: obm-l@mat.puc-rio.br
Assunto: RES: [obm-l] Re: [obm-l] Soma de raízes quadradas

Escreve para esse email

nicolau[AT]mat.puc-rio.br ou nicolau.saldanha[AT]gmail.com

dizendo que quer sair da lista

Enviado do Email<https://go.microsoft.com/fwlink/?LinkId=550986> para Windows 10

De: Lorena Luna<mailto:lorilori20102...@hotmail.com>
Enviado:terça-feira, 18 de fevereiro de 2020 03:22
Para: obm-l@mat.puc-rio.br<mailto:obm-l@mat.puc-rio.br>
Assunto: [obm-l] Re: [obm-l] Soma de raízes quadradas

CANCELAR LISTA DE E-MAIL (Cancelar recebimento)

Em seg, 17 de fev de 2020 às 13:25, Vanderlei Nemitz 
mailto:vanderma...@gmail.com>> escreveu:
Boa tarde!
Existe uma fórmula fechada para a soma das raízes quadradas dos n primeiros 
números naturais?

Muito obrigado!

--
Esta mensagem foi verificada pelo sistema de antiv?rus e
acredita-se estar livre de perigo.

--
Esta mensagem foi verificada pelo sistema de antiv?rus e
acredita-se estar livre de perigo.


--
Esta mensagem foi verificada pelo sistema de antiv?rus e
acredita-se estar livre de perigo.

-- 
Esta mensagem foi verificada pelo sistema de antivírus e
 acredita-se estar livre de perigo.



RES: [obm-l] Re: [obm-l] Soma de raízes quadradas

2020-02-18 Por tôpico Hermann
Escreve para esse email

nicolaumat.puc-rio.br ou nicolau.saldanhagmail.com

dizendo que quer sair da lista

Enviado do Email para Windows 10

De: Lorena Luna
Enviado:terça-feira, 18 de fevereiro de 2020 03:22
Para: obm-l@mat.puc-rio.br
Assunto: [obm-l] Re: [obm-l] Soma de raízes quadradas

CANCELAR LISTA DE E-MAIL (Cancelar recebimento)

Em seg, 17 de fev de 2020 às 13:25, Vanderlei Nemitz  
escreveu:
Boa tarde!
Existe uma fórmula fechada para a soma das raízes quadradas dos n primeiros 
números naturais?

Muito obrigado!

-- 
Esta mensagem foi verificada pelo sistema de antiv�rus e 
acredita-se estar livre de perigo. 

-- 
Esta mensagem foi verificada pelo sistema de antiv�rus e 
acredita-se estar livre de perigo. 


-- 
Esta mensagem foi verificada pelo sistema de antiv�rus e
 acredita-se estar livre de perigo.



[obm-l] Re: [obm-l] Soma de raízes quadradas

2020-02-17 Por tôpico Lorena Luna
CANCELAR LISTA DE E-MAIL (Cancelar recebimento)

Em seg, 17 de fev de 2020 às 13:25, Vanderlei Nemitz 
escreveu:

> Boa tarde!
> Existe uma fórmula fechada para a soma das raízes quadradas dos n
> primeiros números naturais?
>
> Muito obrigado!
>
> --
> Esta mensagem foi verificada pelo sistema de antiv�rus e
> acredita-se estar livre de perigo.

-- 
Esta mensagem foi verificada pelo sistema de antiv�rus e
 acredita-se estar livre de perigo.



[obm-l] Soma de raízes quadradas

2020-02-17 Por tôpico Vanderlei Nemitz
Boa tarde!
Existe uma fórmula fechada para a soma das raízes quadradas dos n primeiros
números naturais?

Muito obrigado!

-- 
Esta mensagem foi verificada pelo sistema de antiv�rus e
 acredita-se estar livre de perigo.



[obm-l] Re: [obm-l] Re: [obm-l] Re: [obm-l] Soma de raízes cúbicas de cossenos

2020-01-24 Por tôpico saulo nilson
NAO PRECISAVA ENCONTRAR COS5, COS 30=COS3*10, DAÍ ENCONTRA O COS10, DEPOIS
É SÓ SUBSTITUIR.

On Fri, Jan 24, 2020 at 10:23 AM Vanderlei Nemitz 
wrote:

> Como?
>
> Não entendi a ideia...
>
>
> Em sex, 24 de jan de 2020 02:37, saulo nilson 
> escreveu:
>
>> COS 15=COS 30/2
>> COS 15=COS(3*5)
>> DAÍ ENCONTRA O VALOR DE COS5 =COS10/2
>> DAÍ ENCONTRA O VALOR DE COS 10
>>
>> S= F(COS 10) QUE ENCONTRA O VALOR
>>
>> On Sun, Jan 19, 2020 at 8:41 AM Vanderlei Nemitz 
>> wrote:
>>
>>> Bom dia, pessoal!
>>>
>>> Pensei em resolver a seguinte questão associando cos 40°, cos 80° e cos
>>> 160° às raízes da equação cos(3x) = -1/2 e utilizando o arco triplo,
>>> recaindo em uma equação de grau 3. Porém, fica difícil determinar o produto
>>> de 2 em 2 das raízes cúbicas. Alguém conhece uma solução melhor?
>>> Muito obrigado!
>>>
>>> S = (cos 40°)^(1/3) + (cos 80°)^(1/3) +  (cos 160°)^(1/3)
>>>
>>> (Soma das raízes cúbicas de cos 40°, cos 80° e cos 160°)
>>>
>>> --
>>> Esta mensagem foi verificada pelo sistema de antivírus e
>>> acredita-se estar livre de perigo.
>>
>>
>> --
>> Esta mensagem foi verificada pelo sistema de antivírus e
>> acredita-se estar livre de perigo.
>
>
> --
> Esta mensagem foi verificada pelo sistema de antivírus e
> acredita-se estar livre de perigo.

-- 
Esta mensagem foi verificada pelo sistema de antiv�rus e
 acredita-se estar livre de perigo.



[obm-l] Re: [obm-l] Re: [obm-l] Soma de raízes cúbicas de cossenos

2020-01-24 Por tôpico Vanderlei Nemitz
Como?

Não entendi a ideia...


Em sex, 24 de jan de 2020 02:37, saulo nilson 
escreveu:

> COS 15=COS 30/2
> COS 15=COS(3*5)
> DAÍ ENCONTRA O VALOR DE COS5 =COS10/2
> DAÍ ENCONTRA O VALOR DE COS 10
>
> S= F(COS 10) QUE ENCONTRA O VALOR
>
> On Sun, Jan 19, 2020 at 8:41 AM Vanderlei Nemitz 
> wrote:
>
>> Bom dia, pessoal!
>>
>> Pensei em resolver a seguinte questão associando cos 40°, cos 80° e cos
>> 160° às raízes da equação cos(3x) = -1/2 e utilizando o arco triplo,
>> recaindo em uma equação de grau 3. Porém, fica difícil determinar o produto
>> de 2 em 2 das raízes cúbicas. Alguém conhece uma solução melhor?
>> Muito obrigado!
>>
>> S = (cos 40°)^(1/3) + (cos 80°)^(1/3) +  (cos 160°)^(1/3)
>>
>> (Soma das raízes cúbicas de cos 40°, cos 80° e cos 160°)
>>
>> --
>> Esta mensagem foi verificada pelo sistema de antivírus e
>> acredita-se estar livre de perigo.
>
>
> --
> Esta mensagem foi verificada pelo sistema de antivírus e
> acredita-se estar livre de perigo.

-- 
Esta mensagem foi verificada pelo sistema de antiv�rus e
 acredita-se estar livre de perigo.



[obm-l] Re: [obm-l] Soma de raízes cúbicas de cossenos

2020-01-23 Por tôpico saulo nilson
COS 15=COS 30/2
COS 15=COS(3*5)
DAÍ ENCONTRA O VALOR DE COS5 =COS10/2
DAÍ ENCONTRA O VALOR DE COS 10

S= F(COS 10) QUE ENCONTRA O VALOR

On Sun, Jan 19, 2020 at 8:41 AM Vanderlei Nemitz 
wrote:

> Bom dia, pessoal!
>
> Pensei em resolver a seguinte questão associando cos 40°, cos 80° e cos
> 160° às raízes da equação cos(3x) = -1/2 e utilizando o arco triplo,
> recaindo em uma equação de grau 3. Porém, fica difícil determinar o produto
> de 2 em 2 das raízes cúbicas. Alguém conhece uma solução melhor?
> Muito obrigado!
>
> S = (cos 40°)^(1/3) + (cos 80°)^(1/3) +  (cos 160°)^(1/3)
>
> (Soma das raízes cúbicas de cos 40°, cos 80° e cos 160°)
>
> --
> Esta mensagem foi verificada pelo sistema de antivírus e
> acredita-se estar livre de perigo.

-- 
Esta mensagem foi verificada pelo sistema de antiv�rus e
 acredita-se estar livre de perigo.



Re: Re: [obm-l] soma com cevianas que passam pelo circuncentro

2020-01-20 Por tôpico qedtexte
Sauda,c~oes, 

Essa frmula no vale para todos os tringulos obtusngulos. 
Daria para caracterizar os tringulos obtusngulos para os 
quais ela  verdadeira ? 

Abraos, 
Lus 



--
Esta mensagem foi verificada pelo sistema de antiv�rus e
acredita-se estar livre de perigo.



[obm-l] Soma de raízes cúbicas de cossenos

2020-01-19 Por tôpico Vanderlei Nemitz
Bom dia, pessoal!

Pensei em resolver a seguinte questão associando cos 40°, cos 80° e cos
160° às raízes da equação cos(3x) = -1/2 e utilizando o arco triplo,
recaindo em uma equação de grau 3. Porém, fica difícil determinar o produto
de 2 em 2 das raízes cúbicas. Alguém conhece uma solução melhor?
Muito obrigado!

S = (cos 40°)^(1/3) + (cos 80°)^(1/3) +  (cos 160°)^(1/3)

(Soma das raízes cúbicas de cos 40°, cos 80° e cos 160°)

-- 
Esta mensagem foi verificada pelo sistema de antiv�rus e
 acredita-se estar livre de perigo.



Re: [obm-l] Uma soma

2020-01-17 Por tôpico Projeto Iteano
Esse problema dá pra resolver usando notação de somatório .
Como eu não sei escrever isso no teclado, eu vou usar E(i=1,n)[x] como
sendo o somatório com i indo de 1 até n de x. Tudo que vier depois do ]
está fora do somatório.
1+(1+2)+...+(1+2+...+n) é o somatório das somas entre os n primeiros
naturais,e portanto igual a:
E(i=1,n)[(i+1)i/2] ,pela fórmula da soma dos termos de uma p.a.
=E (i=1,n)[(i^2+i)]/2
=E(i=1,n)[i^2]/2+E(i=1,n)[i]/2 ,que pela fórmula da soma dos n primeiros
quadrados e dos termos de uma p.a.  é igual a:
[n(n+1)(2n+1)/12 ]+n(n+1)/4
=[n(n+1)(2n+1)+3n(n+1)]/12
=n(n+1)(2n+4)/12
=n(n+1)(n+2)/6  ■


Em Sex, 17 de jan de 2020 17:25, Ralph Teixeira 
escreveu:

> https://en.wikipedia.org/wiki/Tetrahedral_number
>
> On Thu, Jan 16, 2020 at 6:13 PM marcone augusto araújo borges <
> marconeborge...@hotmail.com> wrote:
>
>> Como calcular 1 + (1+2) + (1+2+3) +... +(1+2+...+n)?
>> --
>> Esta mensagem foi verificada pelo sistema de antivírus e
>> acredita-se estar livre de perigo.
>>
>
> --
> Esta mensagem foi verificada pelo sistema de antivírus e
> acredita-se estar livre de perigo.

-- 
Esta mensagem foi verificada pelo sistema de antiv�rus e
 acredita-se estar livre de perigo.



Re: [obm-l] Uma soma

2020-01-17 Por tôpico Ralph Teixeira
https://en.wikipedia.org/wiki/Tetrahedral_number

On Thu, Jan 16, 2020 at 6:13 PM marcone augusto araújo borges <
marconeborge...@hotmail.com> wrote:

> Como calcular 1 + (1+2) + (1+2+3) +... +(1+2+...+n)?
> --
> Esta mensagem foi verificada pelo sistema de antivírus e
> acredita-se estar livre de perigo.
>

-- 
Esta mensagem foi verificada pelo sistema de antiv�rus e
 acredita-se estar livre de perigo.



RES: [obm-l] Uma soma

2020-01-17 Por tôpico Hermann
Eu não sei fazer, mas pelo teorema das diferenças das diferenças que estudei em 
1996 e não lembro mais, com certeza absoluta é um polinômio de grau 3 em n.
Tipo n(n+a)(n+b)+k algo assim...

Sei que não ajudei muito, mas é de grau 3 com certeza.

Enviado do Email para Windows 10

De: marcone augusto araújo borges
Enviado:quinta-feira, 16 de janeiro de 2020 19:47
Para: obm-l@mat.puc-rio.br
Assunto: [obm-l] Uma soma

Como calcular 1 + (1+2) + (1+2+3) +... +(1+2+...+n)? 
-- 
Esta mensagem foi verificada pelo sistema de antivírus e 
acredita-se estar livre de perigo. 


-- 
Esta mensagem foi verificada pelo sistema de antiv�rus e
 acredita-se estar livre de perigo.



Re: [obm-l] Uma soma

2020-01-16 Por tôpico Claudio Buffara
O termo geral é k*(n+1-k), com k variando de 1 a n

Enviado do meu iPhone

> Em 16 de jan de 2020, à(s) 17:27, Claudio Buffara  
> escreveu:
> 
> Faz uma tabela
> 1
> 1   2
> 1   2   3
> 1   2   3  4
> 
> 4*1 + 3*2 + 2*3 + 1*4
> 
> Deu pra pegar o padrão?
> 
> Enviado do meu iPhone
> 
>> Em 16 de jan de 2020, à(s) 16:13, marcone augusto araújo borges 
>>  escreveu:
>> 
>>  Como calcular 1 + (1+2) + (1+2+3) +... +(1+2+...+n)? 
>> -- 
>> Esta mensagem foi verificada pelo sistema de antivírus e 
>> acredita-se estar livre de perigo.

-- 
Esta mensagem foi verificada pelo sistema de antiv�rus e
 acredita-se estar livre de perigo.


=
Instru��es para entrar na lista, sair da lista e usar a lista em
http://www.mat.puc-rio.br/~obmlistas/obm-l.html
=


Re: [obm-l] Uma soma

2020-01-16 Por tôpico Claudio Buffara
Faz uma tabela
1
1   2
1   2   3
1   2   3  4

4*1 + 3*2 + 2*3 + 1*4

Deu pra pegar o padrão?

Enviado do meu iPhone

> Em 16 de jan de 2020, à(s) 16:13, marcone augusto araújo borges 
>  escreveu:
> 
>  Como calcular 1 + (1+2) + (1+2+3) +... +(1+2+...+n)? 
> -- 
> Esta mensagem foi verificada pelo sistema de antivírus e 
> acredita-se estar livre de perigo.

-- 
Esta mensagem foi verificada pelo sistema de antiv�rus e
 acredita-se estar livre de perigo.


=
Instru��es para entrar na lista, sair da lista e usar a lista em
http://www.mat.puc-rio.br/~obmlistas/obm-l.html
=


Re: [obm-l] Uma soma

2020-01-16 Por tôpico Pierre Minner Denizot
1 é somado n vezes,
2 é somado (n-1) vezes,
i é somado (n-i+1) vezes.

Σ(n-i+1)i
Com i de 1 a n
=
(n+1)Σi - Σi²
Com i de 1 a n
O resto deixo contigo

Em qui, 16 de jan de 2020 18:14, marcone augusto araújo borges <
marconeborge...@hotmail.com> escreveu:

> Como calcular 1 + (1+2) + (1+2+3) +... +(1+2+...+n)?
> --
> Esta mensagem foi verificada pelo sistema de antivírus e
> acredita-se estar livre de perigo.
>

-- 
Esta mensagem foi verificada pelo sistema de antiv�rus e
 acredita-se estar livre de perigo.



[obm-l] Uma soma

2020-01-16 Por tôpico marcone augusto araújo borges
Como calcular 1 + (1+2) + (1+2+3) +... +(1+2+...+n)?

-- 
Esta mensagem foi verificada pelo sistema de antivírus e
 acredita-se estar livre de perigo.



Re: [obm-l] Soma de Riemann

2020-01-15 Por tôpico Luiz Antonio Rodrigues
Olá, Bernardo!
Boa tarde!
Vou acessar os links que você indicou.
Muito obrigado!
Luiz

Em qua, 15 de jan de 2020 1:25 PM, Bernardo Freitas Paulo da Costa <
bernardo...@gmail.com> escreveu:

> On Tue, Jan 14, 2020 at 9:45 PM Claudio Buffara
>  wrote:
> > O artigo é esse aqui:
> >
> https://epocanegocios.globo.com/Informacao/Dilemas/noticia/2014/12/elas-precisam-de-reengenharia.html
> > É de 2014, mas ino que a situação não tenha mudado muito de lá pra cá.
>
> Há algumas tentativas de mudança.  Uma delas, o recém-criado curso de
> Engenharia Matemática da UFRJ.  Inspirado, em parte, da experiência de
> intercâmbio com a Polytechnique e a ENSTA (tanto de professores como
> de alunos), e buscando integrar a sólida formação em matemática e
> ciências básicas com o maior centro de pesquisa em engenharia da
> América Latina, a COPPE.
>
> Para mais detalhes sobre o curso, confiram
>
> https://sites.google.com/matematica.ufrj.br/aplicada/engenharia-matem%C3%A1tica
>
> http://www.im.ufrj.br/index.php/pt/noticias-e-eventos/noticias/247-saiba-mais-sobre-o-novo-curso-engenharia-matematica
>
> E, para quem quiser ler a proposta integral:
> http://www.im.ufrj.br/images/documentos/projeto_engenhariamatematica.pdf
>
> Abraços,
> --
> Bernardo Freitas Paulo da Costa
>
> --
> Esta mensagem foi verificada pelo sistema de antivírus e
>  acredita-se estar livre de perigo.
>
>
> =
> Instru�ões para entrar na lista, sair da lista e usar a lista em
> http://www.mat.puc-rio.br/~obmlistas/obm-l.html
> =
>

-- 
Esta mensagem foi verificada pelo sistema de antiv�rus e
 acredita-se estar livre de perigo.



Re: [obm-l] Soma de Riemann

2020-01-15 Por tôpico Bernardo Freitas Paulo da Costa
On Tue, Jan 14, 2020 at 9:45 PM Claudio Buffara
 wrote:
> O artigo é esse aqui:
> https://epocanegocios.globo.com/Informacao/Dilemas/noticia/2014/12/elas-precisam-de-reengenharia.html
> É de 2014, mas ino que a situação não tenha mudado muito de lá pra cá.

Há algumas tentativas de mudança.  Uma delas, o recém-criado curso de
Engenharia Matemática da UFRJ.  Inspirado, em parte, da experiência de
intercâmbio com a Polytechnique e a ENSTA (tanto de professores como
de alunos), e buscando integrar a sólida formação em matemática e
ciências básicas com o maior centro de pesquisa em engenharia da
América Latina, a COPPE.

Para mais detalhes sobre o curso, confiram
https://sites.google.com/matematica.ufrj.br/aplicada/engenharia-matem%C3%A1tica
http://www.im.ufrj.br/index.php/pt/noticias-e-eventos/noticias/247-saiba-mais-sobre-o-novo-curso-engenharia-matematica

E, para quem quiser ler a proposta integral:
http://www.im.ufrj.br/images/documentos/projeto_engenhariamatematica.pdf

Abraços,
-- 
Bernardo Freitas Paulo da Costa

-- 
Esta mensagem foi verificada pelo sistema de antiv�rus e
 acredita-se estar livre de perigo.


=
Instru��es para entrar na lista, sair da lista e usar a lista em
http://www.mat.puc-rio.br/~obmlistas/obm-l.html
=


Re: [obm-l] Soma de Riemann

2020-01-14 Por tôpico Claudio Buffara
Os livros são estes mesmo.

O artigo é esse aqui:
https://epocanegocios.globo.com/Informacao/Dilemas/noticia/2014/12/elas-precisam-de-reengenharia.html

É de 2014, mas ino que a situação não tenha mudado muito de lá pra cá.

[]s,
Claudio.


On Tue, Jan 14, 2020 at 7:09 PM Luiz Antonio Rodrigues <
rodrigue...@gmail.com> wrote:

> Olá, Claudio!
> Tudo bem?
> Muito obrigado pelas sugestões.
> Eu vi na Amazon os títulos:
>
> A Problem Book in Algebra - Krechmar
>
> Problems in Higher Algebra - Faddeev & Sominskii
>
> São esses?
> O que você disse é verdade, muitas vezes eu recorro aos softwares para
> verificar minhas respostas.
> Eu gostaria bastante de ler o artigo que você citou.
> Muito obrigado!
> Abs.
>
> Em ter, 14 de jan de 2020 5:01 PM, Claudio Buffara <
> claudio.buff...@gmail.com> escreveu:
>
>> Estas somas trigonométricas (e várias outras) são obtidas sem grandes
>> dificuldades, mas com alguma álgebra, usando números complexos.
>>
>> O melhor caminho, a meu ver, seria vc conseguir um daqueles livros russos
>> clássicos - Krechmar ou Faddev-Sominski - que contém coletâneas de
>> problemas resolvidos sobre este tema e muitos outros.
>>
>> Agora, me parece que a habilidade de computar estas somas “na mão”,
>> usando complexos e/ou alguns truques algébricos (ou até mesmo integrais)
>> tem se desvalorizado recentemente devido à existência e ampla
>> disponibilidade de softwares gratuitos tais como o Wolfram Alpha, que
>> calculam qualquer soma dessas.
>>
>> Recentemente li um artigo que toca um pouco neste tema, da necessidade de
>> modernização dos cursos de exatas. Vou procurar e postarei aqui.
>>
>> Abs
>>
>>
>> Enviado do meu iPhone
>>
>> Em 14 de jan de 2020, à(s) 12:07, Luiz Antonio Rodrigues <
>> rodrigue...@gmail.com> escreveu:
>>
>> 
>> Olá, Artur!
>> Tudo bem?
>> Agradeço sua resposta.
>> O problema diz:
>>
>> É dado o somatório de:
>>
>> sen(k*b/n)
>>
>> Onde k varia de 1 até n.
>>
>> Calcule o limite deste somatório dividido por n, quando n tende a
>> infinito.
>>
>> O problema pede que se relacione este limite com uma soma de Riemann.
>>
>> Seguindo a sugestão do Claudio, calculei o somatório dos senos em P.A.
>> Depois eu calculei o limite solicitado.
>> Cheguei n mesma resposta do Claudio, que está correta.
>> Aproveito para pedir uma indicação de material sobre este assunto, que
>> considero bastante interessante.
>> Muito obrigado!
>> Luiz
>>
>>
>> Em ter, 14 de jan de 2020 1:32 AM, Artur Costa Steiner <
>> artur.costa.stei...@gmail.com> escreveu:
>>
>>> Este somatório não é uma soma de Riemann. Seria se fosse
>>>
>>> S(n) = 1/n Soma(k = 1, n) sen(k*b/n).Â
>>>
>>> Mas é S(n) = Soma(k = 1, n) sen(k*b/n). Não se divide por n.
>>>
>>> Tem ceteza de que pelo outro processo vc chegou no seu somatório Ã
>>> expressão correspondente ao caso da soma de Riemann?
>>>
>>> Se fizermos b = pi/2, no seu somatorio temos para todo n que S(n) >
>>> sen(pi/2) = 1.Logo, se o limite com n indo para oo existir, será >= 1.Mas
>>> entrando com b = pi/2 na fórmula da soma de Riemann, obtemos 2/pi < 1.
>>>
>>> Me corrija se eu tiver cometido algum erro.
>>>
>>> Abraços
>>>
>>> Artur
>>>
>>> Em seg, 13 de jan de 2020 18:04, Luiz Antonio Rodrigues <
>>> rodrigue...@gmail.com> escreveu:
>>>
>>>> Olá, Claudio!
>>>> Tudo bem?
>>>> Sim, foi esse resultado que eu achei!
>>>> Muito obrigado pela ajuda!
>>>>
>>>> Em seg, 13 de jan de 2020 5:02 PM, Claudio Buffara <
>>>> claudio.buff...@gmail.com> escreveu:
>>>>
>>>>> É a soma de n retângulos, todos com base 1/n e o k-esimo com altura
>>>>> sen(kb/n): logo, o limite e’ a integral superior (portanto, a integral
>>>>> definida) de sen(bx) no intervalo [0,1].
>>>>>
>>>>> A antiderivada é (-1/b)*cos(bx).
>>>>>
>>>>> Logo, a integral é (1 - cos(b))/b.
>>>>>
>>>>> Enviado do meu iPhone
>>>>>
>>>>> Em 13 de jan de 2020, Ã (s) 07:04, Esdras Muniz <
>>>>> esdrasmunizm...@gmail.com> escreveu:
>>>>>
>>>>> 
>>>>> Esse limite vai ser a integral inferior de sen(x) de 0 a b. Daí,
>>>>> como Sen Ã

Re: [obm-l] Soma de Riemann

2020-01-14 Por tôpico Luiz Antonio Rodrigues
Olá, Claudio!
Tudo bem?
Muito obrigado pelas sugestões.
Eu vi na Amazon os títulos:

A Problem Book in Algebra - Krechmar

Problems in Higher Algebra - Faddeev & Sominskii

São esses?
O que você disse é verdade, muitas vezes eu recorro aos softwares para
verificar minhas respostas.
Eu gostaria bastante de ler o artigo que você citou.
Muito obrigado!
Abs.

Em ter, 14 de jan de 2020 5:01 PM, Claudio Buffara <
claudio.buff...@gmail.com> escreveu:

> Estas somas trigonométricas (e várias outras) são obtidas sem grandes
> dificuldades, mas com alguma álgebra, usando números complexos.
>
> O melhor caminho, a meu ver, seria vc conseguir um daqueles livros russos
> clássicos - Krechmar ou Faddev-Sominski - que contém coletâneas de
> problemas resolvidos sobre este tema e muitos outros.
>
> Agora, me parece que a habilidade de computar estas somas “na mão”, usando
> complexos e/ou alguns truques algébricos (ou até mesmo integrais) tem se
> desvalorizado recentemente devido à existência e ampla disponibilidade de
> softwares gratuitos tais como o Wolfram Alpha, que calculam qualquer soma
> dessas.
>
> Recentemente li um artigo que toca um pouco neste tema, da necessidade de
> modernização dos cursos de exatas. Vou procurar e postarei aqui.
>
> Abs
>
>
> Enviado do meu iPhone
>
> Em 14 de jan de 2020, à(s) 12:07, Luiz Antonio Rodrigues <
> rodrigue...@gmail.com> escreveu:
>
> 
> Olá, Artur!
> Tudo bem?
> Agradeço sua resposta.
> O problema diz:
>
> É dado o somatório de:
>
> sen(k*b/n)
>
> Onde k varia de 1 até n.
>
> Calcule o limite deste somatório dividido por n, quando n tende a
> infinito.
>
> O problema pede que se relacione este limite com uma soma de Riemann.
>
> Seguindo a sugestão do Claudio, calculei o somatório dos senos em P.A.
> Depois eu calculei o limite solicitado.
> Cheguei n mesma resposta do Claudio, que está correta.
> Aproveito para pedir uma indicação de material sobre este assunto, que
> considero bastante interessante.
> Muito obrigado!
> Luiz
>
>
> Em ter, 14 de jan de 2020 1:32 AM, Artur Costa Steiner <
> artur.costa.stei...@gmail.com> escreveu:
>
>> Este somatório não é uma soma de Riemann. Seria se fosse
>>
>> S(n) = 1/n Soma(k = 1, n) sen(k*b/n).Â
>>
>> Mas é S(n) = Soma(k = 1, n) sen(k*b/n). Não se divide por n.
>>
>> Tem ceteza de que pelo outro processo vc chegou no seu somatório Ã
>> expressão correspondente ao caso da soma de Riemann?
>>
>> Se fizermos b = pi/2, no seu somatorio temos para todo n que S(n) >
>> sen(pi/2) = 1.Logo, se o limite com n indo para oo existir, será >= 1.Mas
>> entrando com b = pi/2 na fórmula da soma de Riemann, obtemos 2/pi < 1.
>>
>> Me corrija se eu tiver cometido algum erro.
>>
>> Abraços
>>
>> Artur
>>
>> Em seg, 13 de jan de 2020 18:04, Luiz Antonio Rodrigues <
>> rodrigue...@gmail.com> escreveu:
>>
>>> Olá, Claudio!
>>> Tudo bem?
>>> Sim, foi esse resultado que eu achei!
>>> Muito obrigado pela ajuda!
>>>
>>> Em seg, 13 de jan de 2020 5:02 PM, Claudio Buffara <
>>> claudio.buff...@gmail.com> escreveu:
>>>
>>>> É a soma de n retângulos, todos com base 1/n e o k-esimo com altura
>>>> sen(kb/n): logo, o limite e’ a integral superior (portanto, a integral
>>>> definida) de sen(bx) no intervalo [0,1].
>>>>
>>>> A antiderivada é (-1/b)*cos(bx).
>>>>
>>>> Logo, a integral é (1 - cos(b))/b.
>>>>
>>>> Enviado do meu iPhone
>>>>
>>>> Em 13 de jan de 2020, Ã (s) 07:04, Esdras Muniz <
>>>> esdrasmunizm...@gmail.com> escreveu:
>>>>
>>>> 
>>>> Esse limite vai ser a integral inferior de sen(x) de 0 a b. Daí,
>>>> como Sen é integravel, esse limite vai ser Sen(b).
>>>>
>>>> Em dom, 12 de jan de 2020 19:19, Luiz Antonio Rodrigues <
>>>> rodrigue...@gmail.com> escreveu:
>>>>
>>>>> Olá, pessoal!
>>>>> Tudo bem?
>>>>> Estou pensando neste problema há vários dias e não consigo
>>>>> descobrir onde está meu erro.
>>>>> Alguém pode me ajudar?
>>>>>
>>>>> O problema é o seguinte:
>>>>>
>>>>> É dado o somatório de:
>>>>>
>>>>> sen(k*b/n)
>>>>>
>>>>> Onde k varia de 1 até n.
>>>>>
>>>>> Preciso calcular o limite deste somatório divid

Re: [obm-l] Soma de Riemann

2020-01-14 Por tôpico Claudio Buffara
Estas somas trigonométricas (e várias outras) são obtidas sem grandes 
dificuldades, mas com alguma álgebra, usando números complexos. 

O melhor caminho, a meu ver, seria vc conseguir um daqueles livros russos 
clássicos - Krechmar ou Faddev-Sominski - que contém coletâneas de problemas 
resolvidos sobre este tema e muitos outros.

Agora, me parece que a habilidade de computar estas somas “na mão”, usando 
complexos e/ou alguns truques algébricos (ou até mesmo integrais) tem se 
desvalorizado recentemente devido à existência e ampla disponibilidade de 
softwares gratuitos tais como o Wolfram Alpha, que calculam qualquer soma 
dessas.

Recentemente li um artigo que toca um pouco neste tema, da necessidade de 
modernização dos cursos de exatas. Vou procurar e postarei aqui.

Abs


Enviado do meu iPhone

> Em 14 de jan de 2020, à(s) 12:07, Luiz Antonio Rodrigues 
>  escreveu:
> 
> Olá, Artur!
> Tudo bem?
> Agradeço sua resposta.
> O problema diz:
> 
> É dado o somatório de:
> 
> sen(k*b/n)
> 
> Onde k varia de 1 até n.
> 
> Calcule o limite deste somatório dividido por n, quando n tende a infinito.
> 
> O problema pede que se relacione este limite com uma soma de Riemann.
> 
> Seguindo a sugestão do Claudio, calculei o somatório dos senos em P.A.
> Depois eu calculei o limite solicitado.
> Cheguei n mesma resposta do Claudio, que está correta.
> Aproveito para pedir uma indicação de material sobre este assunto, que 
> considero bastante interessante.
> Muito obrigado!
> Luiz
> 
> 
> Em ter, 14 de jan de 2020 1:32 AM, Artur Costa Steiner 
>  escreveu:
>> Este somatório não é uma soma de Riemann. Seria se fosse
>> 
>> S(n) = 1/n Soma(k = 1, n) sen(k*b/n). 
>> 
>> Mas é S(n) = Soma(k = 1, n) sen(k*b/n). Não se divide por n.
>> 
>> Tem ceteza de que pelo outro processo vc chegou no seu somatório à 
>> expressão correspondente ao caso da soma de Riemann?
>> 
>> Se fizermos b = pi/2, no seu somatorio temos para todo n que S(n) > 
>> sen(pi/2) = 1.Logo, se o limite com n indo para oo existir, será >= 1.Mas 
>> entrando com b = pi/2 na fórmula da soma de Riemann, obtemos 2/pi < 1.
>> 
>> Me corrija se eu tiver cometido algum erro.
>> 
>> Abraços
>> 
>> Artur
>> 
>> Em seg, 13 de jan de 2020 18:04, Luiz Antonio Rodrigues 
>>  escreveu:
>>> Olá, Claudio!
>>> Tudo bem?
>>> Sim, foi esse resultado que eu achei!
>>> Muito obrigado pela ajuda!
>>> 
>>> Em seg, 13 de jan de 2020 5:02 PM, Claudio Buffara 
>>>  escreveu:
>>>> É a soma de n retângulos, todos com base 1/n e o k-esimo com altura 
>>>> sen(kb/n): logo, o limite e’ a integral superior (portanto, a integral 
>>>> definida) de sen(bx) no intervalo [0,1].
>>>> 
>>>> A antiderivada é (-1/b)*cos(bx).
>>>> 
>>>> Logo, a integral é (1 - cos(b))/b.
>>>> 
>>>> Enviado do meu iPhone
>>>> 
>>>>> Em 13 de jan de 2020, Ã (s) 07:04, Esdras Muniz 
>>>>>  escreveu:
>>>>> 
>>>>> Esse limite vai ser a integral inferior de sen(x) de 0 a b. Daí, como 
>>>>> Sen é integravel, esse limite vai ser Sen(b).
>>>>> 
>>>>> Em dom, 12 de jan de 2020 19:19, Luiz Antonio Rodrigues 
>>>>>  escreveu:
>>>>>> Olá, pessoal!
>>>>>> Tudo bem?
>>>>>> Estou pensando neste problema há vários dias e não consigo 
>>>>>> descobrir onde está meu erro.
>>>>>> Alguém pode me ajudar?
>>>>>> 
>>>>>> O problema é o seguinte:
>>>>>> 
>>>>>> É dado o somatório de:
>>>>>> 
>>>>>> sen(k*b/n)
>>>>>> 
>>>>>> Onde k varia de 1 até n.
>>>>>> 
>>>>>> Preciso calcular o limite deste somatório dividido por n, quando n 
>>>>>> tende a infinito.
>>>>>> 
>>>>>> O problema pede que se relacione este limite com uma soma de Riemann.
>>>>>> 
>>>>>> Eu cheguei no valor zero, que está errado.
>>>>>> O problema parece simples...
>>>>>> Agradeço desde já!
>>>>>> Luiz
>>>>>> 
>>>>>> 
>>>>>> -- 
>>>>>> Esta mensagem foi verificada pelo sistema de antivírus e 
>>>>>> acredita-se estar livre de perigo.
>>>>> 
>>>>> -- 
>>>>> Esta mensagem foi verificada pelo sistema de antivírus e 
>>>>> acredita-se estar livre de perigo.
>>>> 
>>>> -- 
>>>> Esta mensagem foi verificada pelo sistema de antivírus e 
>>>> acredita-se estar livre de perigo.
>>> 
>>> -- 
>>> Esta mensagem foi verificada pelo sistema de antivírus e 
>>> acredita-se estar livre de perigo.
>> 
>> -- 
>> Esta mensagem foi verificada pelo sistema de antivírus e 
>> acredita-se estar livre de perigo.
> 
> -- 
> Esta mensagem foi verificada pelo sistema de antivírus e 
> acredita-se estar livre de perigo.

-- 
Esta mensagem foi verificada pelo sistema de antiv�rus e
 acredita-se estar livre de perigo.



Re: [obm-l] Soma de Riemann

2020-01-14 Por tôpico Luiz Antonio Rodrigues
Olá, Artur!
Tudo bem?
Agradeço sua resposta.
O problema diz:

É dado o somatório de:

sen(k*b/n)

Onde k varia de 1 até n.

Calcule o limite deste somatório dividido por n, quando n tende a infinito.

O problema pede que se relacione este limite com uma soma de Riemann.

Seguindo a sugestão do Claudio, calculei o somatório dos senos em P.A.
Depois eu calculei o limite solicitado.
Cheguei n mesma resposta do Claudio, que está correta.
Aproveito para pedir uma indicação de material sobre este assunto, que
considero bastante interessante.
Muito obrigado!
Luiz


Em ter, 14 de jan de 2020 1:32 AM, Artur Costa Steiner <
artur.costa.stei...@gmail.com> escreveu:

> Este somatório não é uma soma de Riemann. Seria se fosse
>
> S(n) = 1/n Soma(k = 1, n) sen(k*b/n).
>
> Mas é S(n) = Soma(k = 1, n) sen(k*b/n). Não se divide por n.
>
> Tem ceteza de que pelo outro processo vc chegou no seu somatório à
> expressão correspondente ao caso da soma de Riemann?
>
> Se fizermos b = pi/2, no seu somatorio temos para todo n que S(n) >
> sen(pi/2) = 1.Logo, se o limite com n indo para oo existir, será >= 1.Mas
> entrando com b = pi/2 na fórmula da soma de Riemann, obtemos 2/pi < 1.
>
> Me corrija se eu tiver cometido algum erro.
>
> Abraços
>
> Artur
>
> Em seg, 13 de jan de 2020 18:04, Luiz Antonio Rodrigues <
> rodrigue...@gmail.com> escreveu:
>
>> Olá, Claudio!
>> Tudo bem?
>> Sim, foi esse resultado que eu achei!
>> Muito obrigado pela ajuda!
>>
>> Em seg, 13 de jan de 2020 5:02 PM, Claudio Buffara <
>> claudio.buff...@gmail.com> escreveu:
>>
>>> É a soma de n retângulos, todos com base 1/n e o k-esimo com altura
>>> sen(kb/n): logo, o limite e’ a integral superior (portanto, a integral
>>> definida) de sen(bx) no intervalo [0,1].
>>>
>>> A antiderivada é (-1/b)*cos(bx).
>>>
>>> Logo, a integral é (1 - cos(b))/b.
>>>
>>> Enviado do meu iPhone
>>>
>>> Em 13 de jan de 2020, à(s) 07:04, Esdras Muniz <
>>> esdrasmunizm...@gmail.com> escreveu:
>>>
>>> 
>>> Esse limite vai ser a integral inferior de sen(x) de 0 a b. Daí, como
>>> Sen é integravel, esse limite vai ser Sen(b).
>>>
>>> Em dom, 12 de jan de 2020 19:19, Luiz Antonio Rodrigues <
>>> rodrigue...@gmail.com> escreveu:
>>>
>>>> Olá, pessoal!
>>>> Tudo bem?
>>>> Estou pensando neste problema há vários dias e não consigo descobrir
>>>> onde está meu erro.
>>>> Alguém pode me ajudar?
>>>>
>>>> O problema é o seguinte:
>>>>
>>>> É dado o somatório de:
>>>>
>>>> sen(k*b/n)
>>>>
>>>> Onde k varia de 1 até n.
>>>>
>>>> Preciso calcular o limite deste somatório dividido por n, quando n
>>>> tende a infinito.
>>>>
>>>> O problema pede que se relacione este limite com uma soma de Riemann.
>>>>
>>>> Eu cheguei no valor zero, que está errado.
>>>> O problema parece simples...
>>>> Agradeço desde já!
>>>> Luiz
>>>>
>>>>
>>>> --
>>>> Esta mensagem foi verificada pelo sistema de antivírus e
>>>> acredita-se estar livre de perigo.
>>>
>>>
>>> --
>>> Esta mensagem foi verificada pelo sistema de antivírus e
>>> acredita-se estar livre de perigo.
>>>
>>>
>>> --
>>> Esta mensagem foi verificada pelo sistema de antivírus e
>>> acredita-se estar livre de perigo.
>>>
>>
>> --
>> Esta mensagem foi verificada pelo sistema de antivírus e
>> acredita-se estar livre de perigo.
>
>
> --
> Esta mensagem foi verificada pelo sistema de antivírus e
> acredita-se estar livre de perigo.

-- 
Esta mensagem foi verificada pelo sistema de antiv�rus e
 acredita-se estar livre de perigo.



Re: [obm-l] Soma de Riemann

2020-01-13 Por tôpico Artur Costa Steiner
Este somatório não é uma soma de Riemann. Seria se fosse

S(n) = 1/n Soma(k = 1, n) sen(k*b/n).

Mas é S(n) = Soma(k = 1, n) sen(k*b/n). Não se divide por n.

Tem ceteza de que pelo outro processo vc chegou no seu somatório à
expressão correspondente ao caso da soma de Riemann?

Se fizermos b = pi/2, no seu somatorio temos para todo n que S(n) >
sen(pi/2) = 1.Logo, se o limite com n indo para oo existir, será >= 1.Mas
entrando com b = pi/2 na fórmula da soma de Riemann, obtemos 2/pi < 1.

Me corrija se eu tiver cometido algum erro.

Abraços

Artur

Em seg, 13 de jan de 2020 18:04, Luiz Antonio Rodrigues <
rodrigue...@gmail.com> escreveu:

> Olá, Claudio!
> Tudo bem?
> Sim, foi esse resultado que eu achei!
> Muito obrigado pela ajuda!
>
> Em seg, 13 de jan de 2020 5:02 PM, Claudio Buffara <
> claudio.buff...@gmail.com> escreveu:
>
>> É a soma de n retângulos, todos com base 1/n e o k-esimo com altura
>> sen(kb/n): logo, o limite e’ a integral superior (portanto, a integral
>> definida) de sen(bx) no intervalo [0,1].
>>
>> A antiderivada é (-1/b)*cos(bx).
>>
>> Logo, a integral é (1 - cos(b))/b.
>>
>> Enviado do meu iPhone
>>
>> Em 13 de jan de 2020, à(s) 07:04, Esdras Muniz 
>> escreveu:
>>
>> 
>> Esse limite vai ser a integral inferior de sen(x) de 0 a b. Daí, como
>> Sen é integravel, esse limite vai ser Sen(b).
>>
>> Em dom, 12 de jan de 2020 19:19, Luiz Antonio Rodrigues <
>> rodrigue...@gmail.com> escreveu:
>>
>>> Olá, pessoal!
>>> Tudo bem?
>>> Estou pensando neste problema há vários dias e não consigo descobrir
>>> onde está meu erro.
>>> Alguém pode me ajudar?
>>>
>>> O problema é o seguinte:
>>>
>>> É dado o somatório de:
>>>
>>> sen(k*b/n)
>>>
>>> Onde k varia de 1 até n.
>>>
>>> Preciso calcular o limite deste somatório dividido por n, quando n
>>> tende a infinito.
>>>
>>> O problema pede que se relacione este limite com uma soma de Riemann.
>>>
>>> Eu cheguei no valor zero, que está errado.
>>> O problema parece simples...
>>> Agradeço desde já!
>>> Luiz
>>>
>>>
>>> --
>>> Esta mensagem foi verificada pelo sistema de antivírus e
>>> acredita-se estar livre de perigo.
>>
>>
>> --
>> Esta mensagem foi verificada pelo sistema de antivírus e
>> acredita-se estar livre de perigo.
>>
>>
>> --
>> Esta mensagem foi verificada pelo sistema de antivírus e
>> acredita-se estar livre de perigo.
>>
>
> --
> Esta mensagem foi verificada pelo sistema de antivírus e
> acredita-se estar livre de perigo.

-- 
Esta mensagem foi verificada pelo sistema de antiv�rus e
 acredita-se estar livre de perigo.



Re: [obm-l] Soma de Riemann

2020-01-13 Por tôpico Luiz Antonio Rodrigues
Olá, Claudio!
Tudo bem?
Sim, foi esse resultado que eu achei!
Muito obrigado pela ajuda!

Em seg, 13 de jan de 2020 5:02 PM, Claudio Buffara <
claudio.buff...@gmail.com> escreveu:

> É a soma de n retângulos, todos com base 1/n e o k-esimo com altura
> sen(kb/n): logo, o limite e’ a integral superior (portanto, a integral
> definida) de sen(bx) no intervalo [0,1].
>
> A antiderivada é (-1/b)*cos(bx).
>
> Logo, a integral é (1 - cos(b))/b.
>
> Enviado do meu iPhone
>
> Em 13 de jan de 2020, à(s) 07:04, Esdras Muniz 
> escreveu:
>
> 
> Esse limite vai ser a integral inferior de sen(x) de 0 a b. Daí, como Sen
> é integravel, esse limite vai ser Sen(b).
>
> Em dom, 12 de jan de 2020 19:19, Luiz Antonio Rodrigues <
> rodrigue...@gmail.com> escreveu:
>
>> Olá, pessoal!
>> Tudo bem?
>> Estou pensando neste problema há vários dias e não consigo descobrir
>> onde está meu erro.
>> Alguém pode me ajudar?
>>
>> O problema é o seguinte:
>>
>> É dado o somatório de:
>>
>> sen(k*b/n)
>>
>> Onde k varia de 1 até n.
>>
>> Preciso calcular o limite deste somatório dividido por n, quando n tende
>> a infinito.
>>
>> O problema pede que se relacione este limite com uma soma de Riemann.
>>
>> Eu cheguei no valor zero, que está errado.
>> O problema parece simples...
>> Agradeço desde já!
>> Luiz
>>
>>
>> --
>> Esta mensagem foi verificada pelo sistema de antivírus e
>> acredita-se estar livre de perigo.
>
>
> --
> Esta mensagem foi verificada pelo sistema de antivírus e
> acredita-se estar livre de perigo.
>
>
> --
> Esta mensagem foi verificada pelo sistema de antivírus e
> acredita-se estar livre de perigo.
>

-- 
Esta mensagem foi verificada pelo sistema de antiv�rus e
 acredita-se estar livre de perigo.



Re: [obm-l] Soma de Riemann

2020-01-13 Por tôpico Claudio Buffara
É a soma de n retângulos, todos com base 1/n e o k-esimo com altura sen(kb/n): 
logo, o limite e’ a integral superior (portanto, a integral definida) de 
sen(bx) no intervalo [0,1].

A antiderivada é (-1/b)*cos(bx).

Logo, a integral é (1 - cos(b))/b.

Enviado do meu iPhone

> Em 13 de jan de 2020, à(s) 07:04, Esdras Muniz  
> escreveu:
> 
> 
> Esse limite vai ser a integral inferior de sen(x) de 0 a b. Daí, como Sen é 
> integravel, esse limite vai ser Sen(b).
> 
> Em dom, 12 de jan de 2020 19:19, Luiz Antonio Rodrigues 
>  escreveu:
>> Olá, pessoal!
>> Tudo bem?
>> Estou pensando neste problema há vários dias e não consigo descobrir onde 
>> está meu erro.
>> Alguém pode me ajudar?
>> 
>> O problema é o seguinte:
>> 
>> É dado o somatório de:
>> 
>> sen(k*b/n)
>> 
>> Onde k varia de 1 até n.
>> 
>> Preciso calcular o limite deste somatório dividido por n, quando n tende a 
>> infinito.
>> 
>> O problema pede que se relacione este limite com uma soma de Riemann.
>> 
>> Eu cheguei no valor zero, que está errado.
>> O problema parece simples...
>> Agradeço desde já!
>> Luiz
>> 
>> 
>> -- 
>> Esta mensagem foi verificada pelo sistema de antivírus e 
>> acredita-se estar livre de perigo.
> 
> -- 
> Esta mensagem foi verificada pelo sistema de antivírus e 
> acredita-se estar livre de perigo.

-- 
Esta mensagem foi verificada pelo sistema de antiv�rus e
 acredita-se estar livre de perigo.



Re: [obm-l] Soma de Riemann

2020-01-13 Por tôpico Luiz Antonio Rodrigues
Olá, Claudio!
Olá, Esdras!
Tudo bem?
Muito obrigado pela ajuda!
Eu segui a dica do Claudio e calculei o somatório dos senos em P.A.
Depois eu calculei o limite desse somatório dividido por n.
Mas eu cheguei em

(1/b)*(1-cos(b))

O que será que houve?
Esdras,  você considerou o somatório dividido por n?


Em seg, 13 de jan de 2020 9:04 AM, Esdras Muniz 
escreveu:

> Esse limite vai ser a integral inferior de sen(x) de 0 a b. Daí, como Sen
> é integravel, esse limite vai ser Sen(b).
>
> Em dom, 12 de jan de 2020 19:19, Luiz Antonio Rodrigues <
> rodrigue...@gmail.com> escreveu:
>
>> Olá, pessoal!
>> Tudo bem?
>> Estou pensando neste problema há vários dias e não consigo descobrir onde
>> está meu erro.
>> Alguém pode me ajudar?
>>
>> O problema é o seguinte:
>>
>> É dado o somatório de:
>>
>> sen(k*b/n)
>>
>> Onde k varia de 1 até n.
>>
>> Preciso calcular o limite deste somatório dividido por n, quando n tende
>> a infinito.
>>
>> O problema pede que se relacione este limite com uma soma de Riemann.
>>
>> Eu cheguei no valor zero, que está errado.
>> O problema parece simples...
>> Agradeço desde já!
>> Luiz
>>
>>
>> --
>> Esta mensagem foi verificada pelo sistema de antivírus e
>> acredita-se estar livre de perigo.
>
>
> --
> Esta mensagem foi verificada pelo sistema de antivírus e
> acredita-se estar livre de perigo.

-- 
Esta mensagem foi verificada pelo sistema de antiv�rus e
 acredita-se estar livre de perigo.



Re: [obm-l] soma com cevianas que passam pelo circuncentro

2020-01-13 Por tôpico samuel barbosa
Olá, boa tarde.

Uma outra possibilidade:

Se r_a, r_b e r_c são as distâncias de O aos lados e h_a, h_b e h_c são as
alturas, temos

R/AO_a = (h_a-r_a)/h_a = 1 - [BOC]/[ABC].

Somando as três equações equivalentes, obtemos

R/AO_a+R/BO_b+R/CO_c = 3 - ([BOC]+[AOC]+[AOB])/[ABC] = 2.

Abraços
Samuel


Em dom., 12 de jan. de 2020 às 18:06, Anderson Torres <
torres.anderson...@gmail.com> escreveu:

> Em qua., 18 de dez. de 2019 às 20:47, Luís Lopes
>  escreveu:
> >
> > Sauda,c~oes,
> >
> > Sejam AO_a, BO_B e CO_c as cevianas que passam pelo circuncentro.
> > O_a na reta do lado  etc.
> >
> > Como provar que 1/AO_a + 1/BO_b + 1/CO_c = 2/R ?
> >
>
> Uma forma mais ou menos fácil é usando trigonometria. Calcula cada
> segmento como funçao dos ângulos e do raio do círculo, depois faz as
> contas!
>
> > Luís
> >
> >
> > --
> > Esta mensagem foi verificada pelo sistema de antivírus e
> > acredita-se estar livre de perigo.
>
> --
> Esta mensagem foi verificada pelo sistema de antivírus e
>  acredita-se estar livre de perigo.
>
>
> =
> Instru�ões para entrar na lista, sair da lista e usar a lista em
> http://www.mat.puc-rio.br/~obmlistas/obm-l.html
> =
>

-- 
Esta mensagem foi verificada pelo sistema de antiv�rus e
 acredita-se estar livre de perigo.



Re: [obm-l] Soma de Riemann

2020-01-13 Por tôpico Esdras Muniz
Esse limite vai ser a integral inferior de sen(x) de 0 a b. Daí, como Sen é
integravel, esse limite vai ser Sen(b).

Em dom, 12 de jan de 2020 19:19, Luiz Antonio Rodrigues <
rodrigue...@gmail.com> escreveu:

> Olá, pessoal!
> Tudo bem?
> Estou pensando neste problema há vários dias e não consigo descobrir onde
> está meu erro.
> Alguém pode me ajudar?
>
> O problema é o seguinte:
>
> É dado o somatório de:
>
> sen(k*b/n)
>
> Onde k varia de 1 até n.
>
> Preciso calcular o limite deste somatório dividido por n, quando n tende a
> infinito.
>
> O problema pede que se relacione este limite com uma soma de Riemann.
>
> Eu cheguei no valor zero, que está errado.
> O problema parece simples...
> Agradeço desde já!
> Luiz
>
>
> --
> Esta mensagem foi verificada pelo sistema de antivírus e
> acredita-se estar livre de perigo.

-- 
Esta mensagem foi verificada pelo sistema de antiv�rus e
 acredita-se estar livre de perigo.



Re: [obm-l] Soma de Riemann

2020-01-12 Por tôpico Claudio Buffara
Você sabe como somar os senos de arcos cujas medidas formam uma PA?
Use e^(ix) = cos(x) = i*sen(x).

On Sun, Jan 12, 2020 at 7:19 PM Luiz Antonio Rodrigues <
rodrigue...@gmail.com> wrote:

> Olá, pessoal!
> Tudo bem?
> Estou pensando neste problema há vários dias e não consigo descobrir onde
> está meu erro.
> Alguém pode me ajudar?
>
> O problema é o seguinte:
>
> É dado o somatório de:
>
> sen(k*b/n)
>
> Onde k varia de 1 até n.
>
> Preciso calcular o limite deste somatório dividido por n, quando n tende a
> infinito.
>
> O problema pede que se relacione este limite com uma soma de Riemann.
>
> Eu cheguei no valor zero, que está errado.
> O problema parece simples...
> Agradeço desde já!
> Luiz
>
>
> --
> Esta mensagem foi verificada pelo sistema de antivírus e
> acredita-se estar livre de perigo.

-- 
Esta mensagem foi verificada pelo sistema de antiv�rus e
 acredita-se estar livre de perigo.



[obm-l] Soma de Riemann

2020-01-12 Por tôpico Luiz Antonio Rodrigues
Olá, pessoal!
Tudo bem?
Estou pensando neste problema há vários dias e não consigo descobrir onde
está meu erro.
Alguém pode me ajudar?

O problema é o seguinte:

É dado o somatório de:

sen(k*b/n)

Onde k varia de 1 até n.

Preciso calcular o limite deste somatório dividido por n, quando n tende a
infinito.

O problema pede que se relacione este limite com uma soma de Riemann.

Eu cheguei no valor zero, que está errado.
O problema parece simples...
Agradeço desde já!
Luiz

-- 
Esta mensagem foi verificada pelo sistema de antiv�rus e
 acredita-se estar livre de perigo.



Re: [obm-l] soma com cevianas que passam pelo circuncentro

2020-01-12 Por tôpico Anderson Torres
Em qua., 18 de dez. de 2019 às 20:47, Luís Lopes
 escreveu:
>
> Sauda,c~oes,
>
> Sejam AO_a, BO_B e CO_c as cevianas que passam pelo circuncentro.
> O_a na reta do lado  etc.
>
> Como provar que 1/AO_a + 1/BO_b + 1/CO_c = 2/R ?
>

Uma forma mais ou menos fácil é usando trigonometria. Calcula cada
segmento como funçao dos ângulos e do raio do círculo, depois faz as
contas!

> Luís
>
>
> --
> Esta mensagem foi verificada pelo sistema de antivírus e
> acredita-se estar livre de perigo.

-- 
Esta mensagem foi verificada pelo sistema de antiv�rus e
 acredita-se estar livre de perigo.


=
Instru��es para entrar na lista, sair da lista e usar a lista em
http://www.mat.puc-rio.br/~obmlistas/obm-l.html
=


Re: [obm-l] soma com cevianas que passam pelo circuncentro

2019-12-19 Por tôpico Esdras Muniz
Eu tinha feito algo parecido com essa prova 2. Usando o método k.

Em qui, 19 de dez de 2019 14:43, Luís Lopes 
escreveu:

> Sauda,c~oes,
>
> Encontrei um link com a prova:
>
> https://www.cut-the-knot.org/m/Geometry/CeviansThroughCircumcenter.shtml
>
> Esse site é muito bom.
>
> Eu conhecia a prova 3 mas não sabia que o triângulo tinha que ser
> acutângulo.
> Para triângulo retângulo vale também, por verificação direta.
>
> Aí comecei a rever a prova para triângulos obtusângulos e vi que
> havia um problema com (B-C)=90º. Acho que para triângulos obtusângulos
> a igualdade pode valer mas tem que ver para quais casos ela
> não serve. Talvez (B-C) > 90º como (115º,15º,50º) e (B-C) < 90º
> como (105º,45º,30º) satisfazem mas (B-C) = 90º como (120º,30º,30º)
> não satisfaz. Isso precisaria de outra investigação.
>
> Abraços,
> Luís
>
>
>
>
>
> --
> Esta mensagem foi verificada pelo sistema de antivírus e
> acredita-se estar livre de perigo.
>

-- 
Esta mensagem foi verificada pelo sistema de antiv�rus e
 acredita-se estar livre de perigo.



[obm-l] soma com cevianas que passam pelo circuncentro

2019-12-19 Por tôpico Luís Lopes
Sauda,c~oes,

Encontrei um link com a prova:

https://www.cut-the-knot.org/m/Geometry/CeviansThroughCircumcenter.shtml

Esse site é muito bom.

Eu conhecia a prova 3 mas não sabia que o triângulo tinha que ser acutângulo.
Para triângulo retângulo vale também, por verificação direta.

Aí comecei a rever a prova para triângulos obtusângulos e vi que
havia um problema com (B-C)=90º. Acho que para triângulos obtusângulos
a igualdade pode valer mas tem que ver para quais casos ela
não serve. Talvez (B-C) > 90º como (115º,15º,50º) e (B-C) < 90º
como (105º,45º,30º) satisfazem mas (B-C) = 90º como (120º,30º,30º)
não satisfaz. Isso precisaria de outra investigação.

Abraços,
Luís





-- 
Esta mensagem foi verificada pelo sistema de antivírus e
 acredita-se estar livre de perigo.



[obm-l] soma com cevianas que passam pelo circuncentro

2019-12-18 Por tôpico Luís Lopes
Sauda,c~oes,

Sejam AO_a, BO_B e CO_c as cevianas que passam pelo circuncentro.
O_a na reta do lado  etc.

Como provar que 1/AO_a + 1/BO_b + 1/CO_c = 2/R ?

Luís


-- 
Esta mensagem foi verificada pelo sistema de antivírus e
 acredita-se estar livre de perigo.



[obm-l] Re: [obm-l] Distribuição de probabilidade da soma de números arredondados

2019-08-08 Por tôpico Ralph Teixeira
Não sei a resposta, mas a distribuição deve depender de n  Por exemplo,
se n=2, claramente p(100)=1, enquanto se n é muito grande, eu aposto que
p(0)~1 (escolhendo 10 googlelhões de termos, muito provavelmente quase
todos serão menores que 1/2, e portanto eu aposto que todos arredondam para
0, com muita probabilidade).

Será que dá para achar uma recorrência? Acho que deveríamos começar
pensando no problemas mais genérico e mais simples:

"Dividindo o intervalo [a,b] em dois pedaços, medindo cada pedaço,
arredondando e somando, qual a distribuição de probabilidade da soma S?"
(Aliás, acho que melhor ainda seria perguntar a distribuição de S-(b-a),
isto é, ver o quanto você "ganha" pu "perde" quando subdivide um intervalo.
Acho que isso só depende da parte fracionária de b-a, mas tem que ver
direitinho.)

Afinal, passar de n para n+1 amostras significa escolher um ponto a mais, o
que se resume mais ou menos a esta questão anterior -- bom tem que pesar a
probabilidade de cair em cada intervalo antigo, mas talvez saia algo
pensando assim.

Abraço, Ralph.

On Wed, Aug 7, 2019 at 2:31 PM Rodrigo Ângelo 
wrote:

> Vi o seguinte prolbema num outro grupo que faço parte, e como não teve
> solução por lá, resolvi trazer pra esta lista (irei postar tradução livre
> feita por mim abaixo)
>
> F(n) is the random variable received by partitioning 100 into n parts,
>> rounding those parts, and adding the results. An example partition would
>> be: 49.7, 49.7, 0.6, which rounded becomes 50, 50, 1, added becomes 101.
>> The partition is created by choosing n-1 real numbers in [0,100] uniformly,
>> which implicitly defines a partition. What is the distribution of F(n)?
>
>
> Seja F(n) uma variável aleatória definida particionando o número 100 em n
> partes, arredondando essas partes e adicionando os resultados do
> arredondamento. Um exemplo seria 49,7; 49,7; 0,6; que arredondando fica
> 50; 50; 1; resultando em 101. A partição é criada escolhendo n-1 números
> reais no intervalo [0,100] com distribuição uniforme, que implicitamente
> define uma partição. Qual a distribuição de F(n)?
>
> No exemplo anterior, temos n=3 e os n-1 números sorteados foram 49,7 e
> 99,4.
>
> O arredondamento é feito de forma a minimizar a distância até o inteiro
> mais próximo.
>
> Casos em que o inteiro antecessor e o sucessor são equidistantes (ex: 2,5)
> podem ser desconsiderados, porque têm probabilidade zero.
>
> Casos em que um número é sorteado mais de uma vez também tem probabilidade
> zero.
>
> Fiz uma simulação
> https://drigoangelo.shinyapps.io/MonteCarlo_RoundProblem/ e aparentemente
> a função de probabilidade de F seria aproximadamente (independente de n):
>
> p(100) = 0,600
> p(99) = p(101) = 0,196
> p(98) = p(102) = 0,200
> p(F) = 0 para F não pertencente a {98, 99, 100, 101, 102}.
>
> Não consegui encontrar uma distribuição para F analiticamente, usando a
> definição de fdp. O caminho que eu tentei foi usar que cada número pode ser
> arredondado para cima com distribuição Bernoulli(0,5), mas não consegui
> avançar depois disso.
>
> Atenciosamente,
> Rodrigo de Castro Ângelo
>
> --
> Esta mensagem foi verificada pelo sistema de antivírus e
> acredita-se estar livre de perigo.

-- 
Esta mensagem foi verificada pelo sistema de antiv�rus e
 acredita-se estar livre de perigo.



[obm-l] Re: Distribuição de probabilidade da soma de números arredondados

2019-08-07 Por tôpico Rodrigo Ângelo
Ops, apenas corrigindo a função de probabilidade encontrada por simulação:

p(98) = p(102) = 0,002 (e não 0,200 como estava no e-mail anterior)


Atenciosamente,
Rodrigo de Castro Ângelo


Em qua, 7 de ago de 2019 às 14:20, Rodrigo Ângelo 
escreveu:

> Vi o seguinte prolbema num outro grupo que faço parte, e como não teve
> solução por lá, resolvi trazer pra esta lista (irei postar tradução livre
> feita por mim abaixo)
>
> F(n) is the random variable received by partitioning 100 into n parts,
>> rounding those parts, and adding the results. An example partition would
>> be: 49.7, 49.7, 0.6, which rounded becomes 50, 50, 1, added becomes 101.
>> The partition is created by choosing n-1 real numbers in [0,100] uniformly,
>> which implicitly defines a partition. What is the distribution of F(n)?
>
>
> Seja F(n) uma variável aleatória definida particionando o número 100 em n
> partes, arredondando essas partes e adicionando os resultados do
> arredondamento. Um exemplo seria 49,7; 49,7; 0,6; que arredondando fica
> 50; 50; 1; resultando em 101. A partição é criada escolhendo n-1 números
> reais no intervalo [0,100] com distribuição uniforme, que implicitamente
> define uma partição. Qual a distribuição de F(n)?
>
> No exemplo anterior, temos n=3 e os n-1 números sorteados foram 49,7 e
> 99,4.
>
> O arredondamento é feito de forma a minimizar a distância até o inteiro
> mais próximo.
>
> Casos em que o inteiro antecessor e o sucessor são equidistantes (ex: 2,5)
> podem ser desconsiderados, porque têm probabilidade zero.
>
> Casos em que um número é sorteado mais de uma vez também tem probabilidade
> zero.
>
> Fiz uma simulação
> https://drigoangelo.shinyapps.io/MonteCarlo_RoundProblem/ e aparentemente
> a função de probabilidade de F seria aproximadamente (independente de n):
>
> p(100) = 0,600
> p(99) = p(101) = 0,196
> p(98) = p(102) = 0,200
> p(F) = 0 para F não pertencente a {98, 99, 100, 101, 102}.
>
> Não consegui encontrar uma distribuição para F analiticamente, usando a
> definição de fdp. O caminho que eu tentei foi usar que cada número pode ser
> arredondado para cima com distribuição Bernoulli(0,5), mas não consegui
> avançar depois disso.
>
> Atenciosamente,
> Rodrigo de Castro Ângelo
>

-- 
Esta mensagem foi verificada pelo sistema de antiv�rus e
 acredita-se estar livre de perigo.



[obm-l] Distribuição de probabilidade da soma de números arredondados

2019-08-07 Por tôpico Rodrigo Ângelo
Vi o seguinte prolbema num outro grupo que faço parte, e como não teve
solução por lá, resolvi trazer pra esta lista (irei postar tradução livre
feita por mim abaixo)

F(n) is the random variable received by partitioning 100 into n parts,
> rounding those parts, and adding the results. An example partition would
> be: 49.7, 49.7, 0.6, which rounded becomes 50, 50, 1, added becomes 101.
> The partition is created by choosing n-1 real numbers in [0,100] uniformly,
> which implicitly defines a partition. What is the distribution of F(n)?


Seja F(n) uma variável aleatória definida particionando o número 100 em n
partes, arredondando essas partes e adicionando os resultados do
arredondamento. Um exemplo seria 49,7; 49,7; 0,6; que arredondando fica 50;
50; 1; resultando em 101. A partição é criada escolhendo n-1 números reais
no intervalo [0,100] com distribuição uniforme, que implicitamente define
uma partição. Qual a distribuição de F(n)?

No exemplo anterior, temos n=3 e os n-1 números sorteados foram 49,7 e 99,4.

O arredondamento é feito de forma a minimizar a distância até o inteiro
mais próximo.

Casos em que o inteiro antecessor e o sucessor são equidistantes (ex: 2,5)
podem ser desconsiderados, porque têm probabilidade zero.

Casos em que um número é sorteado mais de uma vez também tem probabilidade
zero.

Fiz uma simulação https://drigoangelo.shinyapps.io/MonteCarlo_RoundProblem/ e
aparentemente a função de probabilidade de F seria aproximadamente
(independente de n):

p(100) = 0,600
p(99) = p(101) = 0,196
p(98) = p(102) = 0,200
p(F) = 0 para F não pertencente a {98, 99, 100, 101, 102}.

Não consegui encontrar uma distribuição para F analiticamente, usando a
definição de fdp. O caminho que eu tentei foi usar que cada número pode ser
arredondado para cima com distribuição Bernoulli(0,5), mas não consegui
avançar depois disso.

Atenciosamente,
Rodrigo de Castro Ângelo

-- 
Esta mensagem foi verificada pelo sistema de antiv�rus e
 acredita-se estar livre de perigo.



[obm-l] Primos da forma 4k+1 podem ser escritos como a soma de dois quadrados.

2019-05-14 Por tôpico Pedro José
Boa tarde!
Há um certo tempo, quando me foi indicado um estudo, pelo Cláudio,
https://kconrad.math.uconn.edu/blurbs/ugradnumthy/Zinotes.pdf; fiquei
surpreso com a simplicidade da demonstração do tema em epígrafe e até
sugeri que jamais me esqueceria da demonstração e morri pela boca pois, me
enrolei ao tentar mostrá-la para um jovem.
Tive de visitar o artigo novamente, embora a demonstração esteja correta,
creio que há um engano em uma referência, que não observara da primeira
feita.

p | (x+i)(x−i)
If p is a Gaussian prime, then by (9.2) p | (x+i) or p | (x-i). pág.33

Creio que deveria ser by(6.5).
Lemma 6.5.Let π be prime in Z[i]. For Gaussian integers α1,...,αr, if π |
α1α2···αr then π divides some αj.

Theorem 9.2. A prime p in Z+ is composite in Z[i] if and only if it is a
sum of two squares.

A referência, do artigo, está correta ou não?
Saudações,
PJMS

-- 
Esta mensagem foi verificada pelo sistema de antiv�rus e
 acredita-se estar livre de perigo.



[obm-l] Re: [obm-l] Re: [obm-l] Re: [obm-l] Combinatória (soma de números)

2019-05-04 Por tôpico Claudio Buffara
Sobre o outro tema, a ideia é parear um número cujo k-ésimo algarismo é A
com outro cujo k-ésimo algarismo é (n+1)-A.
No caso de n = 9, parear A com 10-A.

On Sat, May 4, 2019 at 2:26 PM Vanderlei Nemitz 
wrote:

> Pois é, só penso que o raciocínio não é o mesmo, mas talvez eu esteja
> equivocado. Outra coisa, sem querer abusar, já vi em outras questões, mas é
> correto chamar os algarismos de 1 a 9 de "significativos" e o 0 não? Não
> depende da posição? Com certeza, essa era a intenção do autor,
> desconsiderar o zero, creio.
> Mas...
> *...formados com os n primeiros algarismos significativos...*
>
>
> <https://www.avast.com/sig-email?utm_medium=email_source=link_campaign=sig-email_content=webmail>
>  Livre
> de vírus. www.avast.com
> <https://www.avast.com/sig-email?utm_medium=email_source=link_campaign=sig-email_content=webmail>.
> <#m_-8545587947776098710_DAB4FAD8-2DD7-40BB-A1B8-4E2AA1F9FDF2>
>
> Em sáb, 4 de mai de 2019 às 14:12, Claudio Buffara <
> claudio.buff...@gmail.com> escreveu:
>
>> Não vejo porque não. Você vai ter 9!/2 somas iguais a 10.
>>
>>
>> On Sat, May 4, 2019 at 1:51 PM Vanderlei Nemitz 
>> wrote:
>>
>>> Prezados colegas da lista, a seguinte questão é do IME - RJ, do ano de
>>> 1957/1958.
>>> Gostaria de saber se minha resposta está correta, pois fiquei em dúvida
>>> quando forem utilizados os algarismos de 1 a 9, embora a fórmula "funcione".
>>>
>>> *Determinar a expressão da soma de todos os números de n algarismos,
>>> formados com os n primeiros algarismos significativos.  *
>>>
>>> Inicialmente, pensei que se trata das permutações simples dos algarismos
>>> de 1 a n. Por exemplo, se utilizarmos os algarismos de 1 a 5. A soma do
>>> menor com o maior números formados é 12345 + 54321 = 6. Como existem 60
>>> somas iguais a essa, a soma de todos é 6 x 60 = 360. Analogamente,
>>> para n de 1 a 8, temos que a soma é (n + 1)/9 * 99...9 * n!/2 = [(n + 1)! *
>>> (10^n - 1)]/18. Para n = 9 isso funciona, mas não segue a mesma lógica,
>>> pois 10 não é um algarismo. Alguém poderia confirmar se pensei corretamente
>>> e também esse caso isolado?
>>>
>>> Muito obrigado!
>>>
>>> Vanderlei
>>>
>>>
>>>
>>>
>>> <https://www.avast.com/sig-email?utm_medium=email_source=link_campaign=sig-email_content=webmail>
>>>  Livre
>>> de vírus. www.avast.com
>>> <https://www.avast.com/sig-email?utm_medium=email_source=link_campaign=sig-email_content=webmail>.
>>>
>>> <#m_-8545587947776098710_m_-6684176662232703957_m_8245286071011598569_DAB4FAD8-2DD7-40BB-A1B8-4E2AA1F9FDF2>
>>>
>>> --
>>> Esta mensagem foi verificada pelo sistema de antivírus e
>>> acredita-se estar livre de perigo.
>>
>>
>> --
>> Esta mensagem foi verificada pelo sistema de antivírus e
>> acredita-se estar livre de perigo.
>
>
> --
> Esta mensagem foi verificada pelo sistema de antivírus e
> acredita-se estar livre de perigo.

-- 
Esta mensagem foi verificada pelo sistema de antiv�rus e
 acredita-se estar livre de perigo.



[obm-l] Re: [obm-l] Re: [obm-l] Re: [obm-l] Combinatória (soma de números)

2019-05-04 Por tôpico Claudio Buffara
Sim. Que eu saiba, algarismos significativos são do 1 ao 9.
Nomenclatura ruim, até porque o zero pode ser altamente significativo... e
há um outro significado pra essa expressão, relacionado a precisão de
medidas.

On Sat, May 4, 2019 at 2:26 PM Vanderlei Nemitz 
wrote:

> Pois é, só penso que o raciocínio não é o mesmo, mas talvez eu esteja
> equivocado. Outra coisa, sem querer abusar, já vi em outras questões, mas é
> correto chamar os algarismos de 1 a 9 de "significativos" e o 0 não? Não
> depende da posição? Com certeza, essa era a intenção do autor,
> desconsiderar o zero, creio.
> Mas...
> *...formados com os n primeiros algarismos significativos...*
>
>
> <https://www.avast.com/sig-email?utm_medium=email_source=link_campaign=sig-email_content=webmail>
>  Livre
> de vírus. www.avast.com
> <https://www.avast.com/sig-email?utm_medium=email_source=link_campaign=sig-email_content=webmail>.
> <#m_-8545587947776098710_DAB4FAD8-2DD7-40BB-A1B8-4E2AA1F9FDF2>
>
> Em sáb, 4 de mai de 2019 às 14:12, Claudio Buffara <
> claudio.buff...@gmail.com> escreveu:
>
>> Não vejo porque não. Você vai ter 9!/2 somas iguais a 10.
>>
>>
>> On Sat, May 4, 2019 at 1:51 PM Vanderlei Nemitz 
>> wrote:
>>
>>> Prezados colegas da lista, a seguinte questão é do IME - RJ, do ano de
>>> 1957/1958.
>>> Gostaria de saber se minha resposta está correta, pois fiquei em dúvida
>>> quando forem utilizados os algarismos de 1 a 9, embora a fórmula "funcione".
>>>
>>> *Determinar a expressão da soma de todos os números de n algarismos,
>>> formados com os n primeiros algarismos significativos.  *
>>>
>>> Inicialmente, pensei que se trata das permutações simples dos algarismos
>>> de 1 a n. Por exemplo, se utilizarmos os algarismos de 1 a 5. A soma do
>>> menor com o maior números formados é 12345 + 54321 = 6. Como existem 60
>>> somas iguais a essa, a soma de todos é 6 x 60 = 360. Analogamente,
>>> para n de 1 a 8, temos que a soma é (n + 1)/9 * 99...9 * n!/2 = [(n + 1)! *
>>> (10^n - 1)]/18. Para n = 9 isso funciona, mas não segue a mesma lógica,
>>> pois 10 não é um algarismo. Alguém poderia confirmar se pensei corretamente
>>> e também esse caso isolado?
>>>
>>> Muito obrigado!
>>>
>>> Vanderlei
>>>
>>>
>>>
>>>
>>> <https://www.avast.com/sig-email?utm_medium=email_source=link_campaign=sig-email_content=webmail>
>>>  Livre
>>> de vírus. www.avast.com
>>> <https://www.avast.com/sig-email?utm_medium=email_source=link_campaign=sig-email_content=webmail>.
>>>
>>> <#m_-8545587947776098710_m_-6684176662232703957_m_8245286071011598569_DAB4FAD8-2DD7-40BB-A1B8-4E2AA1F9FDF2>
>>>
>>> --
>>> Esta mensagem foi verificada pelo sistema de antivírus e
>>> acredita-se estar livre de perigo.
>>
>>
>> --
>> Esta mensagem foi verificada pelo sistema de antivírus e
>> acredita-se estar livre de perigo.
>
>
> --
> Esta mensagem foi verificada pelo sistema de antivírus e
> acredita-se estar livre de perigo.

-- 
Esta mensagem foi verificada pelo sistema de antiv�rus e
 acredita-se estar livre de perigo.



[obm-l] Re: [obm-l] Re: [obm-l] Combinatória (soma de números)

2019-05-04 Por tôpico Vanderlei Nemitz
Pois é, só penso que o raciocínio não é o mesmo, mas talvez eu esteja
equivocado. Outra coisa, sem querer abusar, já vi em outras questões, mas é
correto chamar os algarismos de 1 a 9 de "significativos" e o 0 não? Não
depende da posição? Com certeza, essa era a intenção do autor,
desconsiderar o zero, creio.
Mas...
*...formados com os n primeiros algarismos significativos...*

<https://www.avast.com/sig-email?utm_medium=email_source=link_campaign=sig-email_content=webmail>
Livre
de vírus. www.avast.com
<https://www.avast.com/sig-email?utm_medium=email_source=link_campaign=sig-email_content=webmail>.
<#DAB4FAD8-2DD7-40BB-A1B8-4E2AA1F9FDF2>

Em sáb, 4 de mai de 2019 às 14:12, Claudio Buffara <
claudio.buff...@gmail.com> escreveu:

> Não vejo porque não. Você vai ter 9!/2 somas iguais a 10.
>
>
> On Sat, May 4, 2019 at 1:51 PM Vanderlei Nemitz 
> wrote:
>
>> Prezados colegas da lista, a seguinte questão é do IME - RJ, do ano de
>> 1957/1958.
>> Gostaria de saber se minha resposta está correta, pois fiquei em dúvida
>> quando forem utilizados os algarismos de 1 a 9, embora a fórmula "funcione".
>>
>> *Determinar a expressão da soma de todos os números de n algarismos,
>> formados com os n primeiros algarismos significativos.  *
>>
>> Inicialmente, pensei que se trata das permutações simples dos algarismos
>> de 1 a n. Por exemplo, se utilizarmos os algarismos de 1 a 5. A soma do
>> menor com o maior números formados é 12345 + 54321 = 6. Como existem 60
>> somas iguais a essa, a soma de todos é 6 x 60 = 360. Analogamente,
>> para n de 1 a 8, temos que a soma é (n + 1)/9 * 99...9 * n!/2 = [(n + 1)! *
>> (10^n - 1)]/18. Para n = 9 isso funciona, mas não segue a mesma lógica,
>> pois 10 não é um algarismo. Alguém poderia confirmar se pensei corretamente
>> e também esse caso isolado?
>>
>> Muito obrigado!
>>
>> Vanderlei
>>
>>
>>
>>
>> <https://www.avast.com/sig-email?utm_medium=email_source=link_campaign=sig-email_content=webmail>
>>  Livre
>> de vírus. www.avast.com
>> <https://www.avast.com/sig-email?utm_medium=email_source=link_campaign=sig-email_content=webmail>.
>>
>> <#m_-6684176662232703957_m_8245286071011598569_DAB4FAD8-2DD7-40BB-A1B8-4E2AA1F9FDF2>
>>
>> --
>> Esta mensagem foi verificada pelo sistema de antivírus e
>> acredita-se estar livre de perigo.
>
>
> --
> Esta mensagem foi verificada pelo sistema de antivírus e
> acredita-se estar livre de perigo.

-- 
Esta mensagem foi verificada pelo sistema de antiv�rus e
 acredita-se estar livre de perigo.



[obm-l] Re: [obm-l] Combinatória (soma de números)

2019-05-04 Por tôpico Claudio Buffara
Não vejo porque não. Você vai ter 9!/2 somas iguais a 10.


On Sat, May 4, 2019 at 1:51 PM Vanderlei Nemitz 
wrote:

> Prezados colegas da lista, a seguinte questão é do IME - RJ, do ano de
> 1957/1958.
> Gostaria de saber se minha resposta está correta, pois fiquei em dúvida
> quando forem utilizados os algarismos de 1 a 9, embora a fórmula "funcione".
>
> *Determinar a expressão da soma de todos os números de n algarismos,
> formados com os n primeiros algarismos significativos.  *
>
> Inicialmente, pensei que se trata das permutações simples dos algarismos
> de 1 a n. Por exemplo, se utilizarmos os algarismos de 1 a 5. A soma do
> menor com o maior números formados é 12345 + 54321 = 6. Como existem 60
> somas iguais a essa, a soma de todos é 6 x 60 = 360. Analogamente,
> para n de 1 a 8, temos que a soma é (n + 1)/9 * 99...9 * n!/2 = [(n + 1)! *
> (10^n - 1)]/18. Para n = 9 isso funciona, mas não segue a mesma lógica,
> pois 10 não é um algarismo. Alguém poderia confirmar se pensei corretamente
> e também esse caso isolado?
>
> Muito obrigado!
>
> Vanderlei
>
>
>
>
> <https://www.avast.com/sig-email?utm_medium=email_source=link_campaign=sig-email_content=webmail>
>  Livre
> de vírus. www.avast.com
> <https://www.avast.com/sig-email?utm_medium=email_source=link_campaign=sig-email_content=webmail>.
> <#m_8245286071011598569_DAB4FAD8-2DD7-40BB-A1B8-4E2AA1F9FDF2>
>
> --
> Esta mensagem foi verificada pelo sistema de antivírus e
> acredita-se estar livre de perigo.

-- 
Esta mensagem foi verificada pelo sistema de antiv�rus e
 acredita-se estar livre de perigo.



[obm-l] Combinatória (soma de números)

2019-05-04 Por tôpico Vanderlei Nemitz
Prezados colegas da lista, a seguinte questão é do IME - RJ, do ano de
1957/1958.
Gostaria de saber se minha resposta está correta, pois fiquei em dúvida
quando forem utilizados os algarismos de 1 a 9, embora a fórmula "funcione".

*Determinar a expressão da soma de todos os números de n algarismos,
formados com os n primeiros algarismos significativos.  *

Inicialmente, pensei que se trata das permutações simples dos algarismos de
1 a n. Por exemplo, se utilizarmos os algarismos de 1 a 5. A soma do menor
com o maior números formados é 12345 + 54321 = 6. Como existem 60 somas
iguais a essa, a soma de todos é 6 x 60 = 360. Analogamente, para n
de 1 a 8, temos que a soma é (n + 1)/9 * 99...9 * n!/2 = [(n + 1)! * (10^n
- 1)]/18. Para n = 9 isso funciona, mas não segue a mesma lógica, pois 10
não é um algarismo. Alguém poderia confirmar se pensei corretamente e
também esse caso isolado?

Muito obrigado!

Vanderlei



<https://www.avast.com/sig-email?utm_medium=email_source=link_campaign=sig-email_content=webmail>
Livre
de vírus. www.avast.com
<https://www.avast.com/sig-email?utm_medium=email_source=link_campaign=sig-email_content=webmail>.
<#DAB4FAD8-2DD7-40BB-A1B8-4E2AA1F9FDF2>

-- 
Esta mensagem foi verificada pelo sistema de antiv�rus e
 acredita-se estar livre de perigo.



[obm-l] Re: [obm-l] Soma de frações próprias

2018-12-27 Por tôpico Anderson Torres
Em sex, 21 de dez de 2018 às 21:09, Daniel Quevedo
 escreveu:
>
> Colocando-se a fração 19/94 sob a forma 1/m + 1/n , onde m e n são inteiros 
> positivos o valor de m + n é igual a:
>

Hum...

1/m+1/n=19/94
(m+n)/(mn)=19/94

94m+94n = 19mn

19mn - 94m = 94n
m(19n-94) = 94n
19m(19n-94) = 94 * 19n
19m(19n-94) = 94 * (19n-94) + 94*94

(19m-94)(19n-94) = 94*94

Daqui já dá para prosseguir com fatorações marotas...

(19(m-5)+1)(19(n-5)+1) = 2*2*47*47

Divisores de 2*2*47*47:
1, 2, 4, 47, 94, 188, 2209, 4418, 8836

Resto 1 módulo 19:
1, 8836

Logo, m=5 e n=470

E m+n= 475, ueba!

> R: 475
> --
> Fiscal: Daniel Quevedo
>
> --
> Esta mensagem foi verificada pelo sistema de antivírus e
> acredita-se estar livre de perigo.

-- 
Esta mensagem foi verificada pelo sistema de antiv�rus e
 acredita-se estar livre de perigo.


=
Instru��es para entrar na lista, sair da lista e usar a lista em
http://www.mat.puc-rio.br/~obmlistas/obm-l.html
=


[obm-l] Re: [obm-l] Soma de frações próprias

2018-12-21 Por tôpico Pacini Bores
 

Oi Daniel, 

Faça (94-19m).(94-19n)=94^2 e  

Abraços 

Pacini 

Em 21/12/2018 21:00, Daniel Quevedo escreveu: 

> Colocando-se a fração 19/94 sob a forma 1/m + 1/n , onde m e n são inteiros 
> positivos o valor de m + n é igual a: 
> 
> R: 475 -- 
> 
> Fiscal: Daniel Quevedo 
> -- 
> Esta mensagem foi verificada pelo sistema de antivrus e 
> acredita-se estar livre de perigo.

 
-- 
Esta mensagem foi verificada pelo sistema de antiv�rus e
 acredita-se estar livre de perigo.



[obm-l] Soma de frações próprias

2018-12-21 Por tôpico Daniel Quevedo
Colocando-se a fração 19/94 sob a forma 1/m + 1/n , onde m e n são inteiros
positivos o valor de m + n é igual a:

R: 475
-- 
Fiscal: Daniel Quevedo

-- 
Esta mensagem foi verificada pelo sistema de antiv�rus e
 acredita-se estar livre de perigo.



Re: [obm-l] Soma de binomiais

2018-11-20 Por tôpico Vanderlei Nemitz
Muito obrigado, Anderson! Vou estudar o artigo.

Em dom, 18 de nov de 2018 09:50, Anderson Torres <
torres.anderson...@gmail.com escreveu:

> Em qua, 7 de nov de 2018 às 14:38, Vanderlei Nemitz
>  escreveu:
> >
> > Boa tarde!
> > Na seguinte questão, tentei pensar no desenvolvimento de algum binômio,
> em que a parte real fosse a soma S(k), mas não consegui imaginar um.
> Fazendo alguns casos, para k de 1 a 4, conjecturei que S(k) = 2^(2k -
> 1).[2^(2k - 1) + (-1)^k].
> > Mas como posso provar que é verdadeira (se realmente for), a partir do
> zero, de preferência sem usar indução?
> >
> > Outra coisa, depois de obtida a fórmula, como obter o menor k que
> satisfaz o problema sem muitas contas? Eu testei até k igual a 14, usando
> uma calculadora.
> >
> > Obrigado!
> >
> > Seja S(k) = C(4k, 0) + C(4k, 4) + C(4k, 8) + ... + C(4k, 4k), para k =
> 1, 2, 3, 
> > O menor valor de k tal que S(k) é múltiplo de 81, é:
> > a) 7
> > b) 9
> > c) 10
> > d) 12
> > e) 14
> >
>
> Cê pode aprender sobre o Método de Multi-Secção no Problema 4 do
> artigo "Raízes da Unidade", na Eureka! 33:
>
> https://www.obm.org.br/content/uploads/2017/01/Eureka33.pdf
>
> > --
> > Esta mensagem foi verificada pelo sistema de antivírus e
> > acredita-se estar livre de perigo.
>
> --
> Esta mensagem foi verificada pelo sistema de antivírus e
>  acredita-se estar livre de perigo.
>
>
> =
> Instru�ões para entrar na lista, sair da lista e usar a lista em
> http://www.mat.puc-rio.br/~obmlistas/obm-l.html
> =
>

-- 
Esta mensagem foi verificada pelo sistema de antiv�rus e
 acredita-se estar livre de perigo.



Re: [obm-l] Soma de binomiais

2018-11-18 Por tôpico Anderson Torres
Em qua, 7 de nov de 2018 às 14:38, Vanderlei Nemitz
 escreveu:
>
> Boa tarde!
> Na seguinte questão, tentei pensar no desenvolvimento de algum binômio, em 
> que a parte real fosse a soma S(k), mas não consegui imaginar um. Fazendo 
> alguns casos, para k de 1 a 4, conjecturei que S(k) = 2^(2k - 1).[2^(2k - 1) 
> + (-1)^k].
> Mas como posso provar que é verdadeira (se realmente for), a partir do zero, 
> de preferência sem usar indução?
>
> Outra coisa, depois de obtida a fórmula, como obter o menor k que satisfaz o 
> problema sem muitas contas? Eu testei até k igual a 14, usando uma 
> calculadora.
>
> Obrigado!
>
> Seja S(k) = C(4k, 0) + C(4k, 4) + C(4k, 8) + ... + C(4k, 4k), para k = 1, 2, 
> 3, 
> O menor valor de k tal que S(k) é múltiplo de 81, é:
> a) 7
> b) 9
> c) 10
> d) 12
> e) 14
>

Cê pode aprender sobre o Método de Multi-Secção no Problema 4 do
artigo "Raízes da Unidade", na Eureka! 33:

https://www.obm.org.br/content/uploads/2017/01/Eureka33.pdf

> --
> Esta mensagem foi verificada pelo sistema de antivírus e
> acredita-se estar livre de perigo.

-- 
Esta mensagem foi verificada pelo sistema de antiv�rus e
 acredita-se estar livre de perigo.


=
Instru��es para entrar na lista, sair da lista e usar a lista em
http://www.mat.puc-rio.br/~obmlistas/obm-l.html
=


[obm-l] Re: [obm-l] Re: [obm-l] Re: [obm-l] [Problema] Achar o mínimo do valor absoluto de uma soma complexa

2018-11-12 Por tôpico Bernardo Freitas Paulo da Costa
Oi,

acho que você interpretou o enunciado de forma a "evitar os
complexos".  O problema original fala de "achar um ponto dentro do
círculo", então talvez não sejam apenas os pontos na circunferência
(como parece que a sua solução faz, ao ordenar todos pelos ângulos
centrais), mas qualquer ponto da forma r*cis(theta).  E daí talvez
tenha mais a ver com complexos...

On Mon, Nov 5, 2018 at 4:51 PM Pedro José  wrote:
>
> Boa tarde!
> Se entendi o que você quer, não entendi qual a relação com o mínimo de uma 
> soma complexa?
> Para resolver o problema que você propõe, entendi:
> (i) a excursão como a geração de um setor circular, a partir de um ponto 
> inicial, essa incursão tem dois sentidos, trigonométrico ou horário.
> (ii) Englobar um ponto significa que o ponto pertença ao setor circular, tem 
> que saber se incluem-se os pontos de borda ou não, como não há restrição vou 
> considerar que sim.
> (iii) Estou supondo que seu universo é plano.
> Minha sugestão é defina o conjunto de pontos em coordenadas polares.
> Defina a variável excursão, e dê a ela um sinal para definir o sentido,
> Faça um programa.
> Definir "arrays" dos pontos (caso não estejam em coordenadas polares, tem que 
> fazer uma sub-rotina para transformar as coordenadas em polares) Mod(I) e 
> Teta(I)
> Defina um array de contagens
> Definir uma rotina para contar o número de pontos. N
> Defina uma sub-rotina Achapontos para determinar o índice Imax, cujo 
> Engloba(I) seja máximo.
> ! Comentário: Atentar que podem retornar mais do que um índice. Portanto 
> deve-se definir um array Pontonotável e uma variável de contagem Nmax. E.g., 
> se tiverem três pontos que englobem o número máximo de pontos, deve retornar: 
> Array contagem, com os valores dos índices dos pontos que têm o máximo de 
> Engloba, nas três primeiras posições e o Valor Nmax=3.
> Aplique a sub-rotina de contagem no array de pontos e retorne com N.
> Entre com o valor de excursão
> Faça de I=1 a N
> Tetamax= max (teta(I);teta(i)+excursão)
> Tetamin=min(teta(I);teta(i)+excursão)
> Engloba(I)=0
> Faça de J=1 a N
> Se (teta(J)<=tetamax e teta(J)>=tetamin e mod(J)<=mod(I).
> ! comentário: A relação engloba será reflexiva. Todo ponto engloba si 
> próprio.Caso não se aceite a borda é só tirar os iguais da lógica acima.
> Engloba(I) = Engloba(I)+1
> Fim SE;
> Fim Faça
> Fim faça
> Aplica Sub-rotina acha pontos.
> Salva temos o máximo de pontos englobados para uma excurção de [excursão] 
> para [Nmax] pontos englobando [engloba(Nmax)]
> São eles:
> Faça de I=1 até N
> Pontonotável(I)
> Fim faça.
> !se tiver interesse salva todo array Engloba.
> FIM.
>
> Porém para qualquer setor existir um ponto que seja sempre o que englobe mais 
> pontos, creio que vá depender da nuvem, e.g.
>
> P1= (10,40)
> P2= ( 6,42)
> P3= (9,90)
> P4= (8,100)
> P5= (7,107)
> P6= (7,5; 108)
> Teremos para uma excursão de + 5 graus:
> Engloba (1) = 1;Engloba (2) = 0; Engloba (3) = 0; Engloba (4) = 0 , Engloba 
> (5) = 0 e Engloba (6) = 0
> P1 é o que engloba mais pontos.
> Para uma excursão de +10 graus:
> Engloba (1) = 1;Engloba (2) = 0; Engloba (3) = 1; Engloba (4) = 2 e Engloba 
> (5) = 0 e Engloba (6) = 0
> P3 é o que engloba mais pontos.
>
> Espero ter compreendido o proposto e ajudado.
> Mas o que tem haver com soma de complexos, módulo mínimo???
>
> Saudações,
> PJMS
>
> Em sáb, 3 de nov de 2018 às 22:31, Bruno Visnadi 
>  escreveu:
>>
>> Não entendi a pergunta - o que é uma excursão?
>>
>> Em sáb, 3 de nov de 2018 às 22:18, Jardiel Cunha  
>> escreveu:
>>>
>>> Olá!
>>>
>>>
>>> Estou trabalhando em um projeto e um problema está me tirando o sono há 
>>> algum tempo. Meu trabalho é na área de engenharia de microondas. A solução 
>>> que eu encontrei até agora, acha soluções mas não satisfatórias... Não 
>>> precisam fazer o problema, queria apenas uma luz em que caminho seguir.
>>>
>>>
>>> [Problema] Dados N pontos em um círculo, estou querendo achar um ponto 
>>> dentro do círculo tal que: para qualquer valor de excursão em graus, eu 
>>> garanta que não existe outro ponto que englobe mais pontos no círculo do 
>>> que ele.
>>>
>>>
>>> Por exemplo: se eu der uma excursão de 80 graus... então eu quero um ponto 
>>> tal que englobe o maior número possível desses N pontos estando ele no 
>>> centro de um arco de 80 graus.
>>>
>>>
>>> Mais um exemplo: tenho 10 pontos. Queria um ponto x tal que ele será o 
>>> centro de todos os arcos com o maior número possível de pontos.
>>>
>>>

Re: [obm-l] Soma de binomiais

2018-11-08 Por tôpico Claudio Buffara
Inicialmente, sabemos que:
A = 1 + C(n,1) + C(n,2) + C(n,3) + ... = 2^n
e
B = 1 + C(n,2) + C(n,4) + ... = C(n,1) + C(n,3) + C(n,5) + ...
(basta expandir (1 + 1)^n  e  (1 - 1)^n).

Além disso:
A - B = C(n,1) + C(n,3) + C(n,5) + ... = B ==> B = A/2 = 2^(n-1)

Também temos:
(1 + i)^n = 1 + C(n,1)*i - C(n,2) - C(n,3)*i + C(n,4) + ...
(1 - i)^n = 1 - C(n,1)*i - C(n,2) + C(n,3)*i + C(n,4) + ...

De modo que:
C = ((1 + i)^n + (1 - i)^n)/2 = 1 - C(n,2) + C(n,4) - C(n,6) + ...

Logo (usando a versão "par" de B):
(B + C)/2 =  1 + C(n,4) + C(n,8) + ... = ( 2^(n-1)  +  ((1 + i)^n + (1 -
i)^n)/2 )/2

Ou seja,
1 + C(n,4) + C(n,8) + ... =  2^(n-2) + ((1 + i)^n + (1 - i)^n)/4(*)

Agora...
(1 + i)^n + (1 - i)^n = 2^(n/2)*(cis(n*pi/4) + cis(-n*pi/4)) = 2^(1 +
n/2)*cos(n*pi/4) ==>
((1 + i)^n + (1 - i)^n)/4 = 2^(n/2 - 1)*cos(n*pi/4)

Se n = 4k, então o lado direito é igual a 2^(2k-1)*cos(k*pi) =
(-1)^k*2^(2k-1)   (**)

Substituindo (**) em (*) e usando que n = 4k, teremos:
1 + C(4k,4) + C(4k,8 ) + ... + C(4k,4k) = 2^(4k-2) + (-1)^k*2^(2k-1) =
2^(2k-1) * (2^(2k-1) + (-1)^k)   (***)

***

Pra (***) ser múltiplo de 81, 2^(2k-1) + (-1)^k terá que ser múltiplo de 81.
Em particular, terá que ser múltiplo de 3.
k = 1 ==> 2^1 - 1 = 1
k = 2 ==> 2^3 + 1 = 9
k = 3 ==> 2^5 - 1 = 31
k = 4 ==> 2^7 + 1 = 129
..
Assim, parece que k = 2m + 2 (m >=0) é condição necessária.
E, de fato, k = 2m+2 ==> 2^(2k-1) + (-1)^k = 2^(4m+3) + 1 = 8*16^m + 1 ==
-1*1 + 1 == 0 (mod 3)   ("==" quer dizer "é congruente a")

Olhando mod 9, teremos 8*16^m + 1 == (-1)*(-2)^m + 1 == 0 (mod 9) sss
(-2)^m == 1 (mod 9).
Isso é verdade pra m = 0, 3, 6 e 9.
Assim, conjecturo que para m = 3p (p>=0) e, portanto, k = 6p + 2 (p >= 0),
2^(2k-1) + (-1)^k = 2^(12p+3) + 1 é múltiplo de 9.
E, de fato, olhando mod 9:  2^(12p+3) +1 == 8*4096^p + 1 == (-1)*1 + 1 == 0
(mod 9)

Testando a divisibilidade por 81:
p = 0 ==> k = 2 ==> 2^3 + 1 ==> não
p = 1 ==> k = 8 ==> 2^15 + 1 = 1024*32 + 1 == 52*32 + 1 = 1665 == 45 ==> não
p = 2 ==> k = 14 ==> 2^27 + 1 = 1024*1024*128 + 1 == 52*52*47 + 1 ==
(-29)*(-29)*(-34) + 1 == 31*(-34) + 1 = -1053 == 0 (mod 81).

Logo, o menor k é 14.

Agora, como é uma múltipla escolha, daria pra ir testando as alternativas
na expressão (***).
Acho que levaria menos tempo.

[]s,
Claudio.


On Wed, Nov 7, 2018 at 2:38 PM Vanderlei Nemitz 
wrote:

> Boa tarde!
> Na seguinte questão, tentei pensar no desenvolvimento de algum binômio, em
> que a parte real fosse a soma S(k), mas não consegui imaginar um. Fazendo
> alguns casos, para k de 1 a 4, conjecturei que S(k) = 2^(2k - 1).[2^(2k -
> 1) + (-1)^k].
> Mas como posso provar que é verdadeira (se realmente for), a partir do
> zero, de preferência sem usar indução?
>
> Outra coisa, depois de obtida a fórmula, como obter o menor k que satisfaz
> o problema sem muitas contas? Eu testei até k igual a 14, usando uma
> calculadora.
>
> Obrigado!
>
> Seja S(k) = C(4k, 0) + C(4k, 4) + C(4k, 8) + ... + C(4k, 4k), para k = 1,
> 2, 3, 
> O menor valor de k tal que S(k) é múltiplo de 81, é:
> a) 7
> b) 9
> c) 10
> d) 12
> e) 14
>
> --
> Esta mensagem foi verificada pelo sistema de antivírus e
> acredita-se estar livre de perigo.

-- 
Esta mensagem foi verificada pelo sistema de antiv�rus e
 acredita-se estar livre de perigo.



[obm-l] Soma de binomiais

2018-11-07 Por tôpico Vanderlei Nemitz
Boa tarde!
Na seguinte questão, tentei pensar no desenvolvimento de algum binômio, em
que a parte real fosse a soma S(k), mas não consegui imaginar um. Fazendo
alguns casos, para k de 1 a 4, conjecturei que S(k) = 2^(2k - 1).[2^(2k -
1) + (-1)^k].
Mas como posso provar que é verdadeira (se realmente for), a partir do
zero, de preferência sem usar indução?

Outra coisa, depois de obtida a fórmula, como obter o menor k que satisfaz
o problema sem muitas contas? Eu testei até k igual a 14, usando uma
calculadora.

Obrigado!

Seja S(k) = C(4k, 0) + C(4k, 4) + C(4k, 8) + ... + C(4k, 4k), para k = 1,
2, 3, 
O menor valor de k tal que S(k) é múltiplo de 81, é:
a) 7
b) 9
c) 10
d) 12
e) 14

-- 
Esta mensagem foi verificada pelo sistema de antiv�rus e
 acredita-se estar livre de perigo.



[obm-l] Re: [obm-l] Re: [obm-l] [Problema] Achar o mínimo do valor absoluto de uma soma complexa

2018-11-05 Por tôpico Pedro José
Boa tarde!

Engano P4 e não Pe é o que engloba mais pontos.
E temos que somar 1 a ca engloba, pois esqueci de contar o próprio ponto.
Mas não influencia para o que englobe o máximo.

Saudações,
PJMS

Em seg, 5 de nov de 2018 às 16:41, Pedro José 
escreveu:

> Boa tarde!
> Se entendi o que você quer, não entendi qual a relação com o mínimo de uma
> soma complexa?
> Para resolver o problema que você propõe, entendi:
> (i) a excursão como a geração de um setor circular, a partir de um ponto
> inicial, essa incursão tem dois sentidos, trigonométrico ou horário.
> (ii) Englobar um ponto significa que o ponto pertença ao setor circular,
> tem que saber se incluem-se os pontos de borda ou não, como não há
> restrição vou considerar que sim.
> (iii) Estou supondo que seu universo é plano.
> Minha sugestão é defina o conjunto de pontos em coordenadas polares.
> Defina a variável excursão, e dê a ela um sinal para definir o sentido,
> Faça um programa.
> Definir "arrays" dos pontos (caso não estejam em coordenadas polares, tem
> que fazer uma sub-rotina para transformar as coordenadas em polares) Mod(I)
> e Teta(I)
> Defina um array de contagens
> Definir uma rotina para contar o número de pontos. N
> Defina uma sub-rotina Achapontos para determinar o índice Imax, cujo
> Engloba(I) seja máximo.
> ! Comentário: Atentar que podem retornar mais do que um índice. Portanto
> deve-se definir um array Pontonotável e uma variável de contagem Nmax.
> E.g., se tiverem três pontos que englobem o número máximo de pontos, deve
> retornar: Array contagem, com os valores dos índices dos pontos que têm o
> máximo de Engloba, nas três primeiras posições e o Valor Nmax=3.
> Aplique a sub-rotina de contagem no array de pontos e retorne com N.
> Entre com o valor de excursão
> Faça de I=1 a N
> Tetamax= max (teta(I);teta(i)+excursão)
> Tetamin=min(teta(I);teta(i)+excursão)
> Engloba(I)=0
> Faça de J=1 a N
> Se (teta(J)<=tetamax e teta(J)>=tetamin e mod(J)<=mod(I).
> ! comentário: A relação engloba será reflexiva. Todo ponto engloba si
> próprio.Caso não se aceite a borda é só tirar os iguais da lógica acima.
> Engloba(I) = Engloba(I)+1
> Fim SE;
> Fim Faça
> Fim faça
> Aplica Sub-rotina acha pontos.
> Salva temos o máximo de pontos englobados para uma excurção de [excursão]
> para [Nmax] pontos englobando [engloba(Nmax)]
> São eles:
> Faça de I=1 até N
> Pontonotável(I)
> Fim faça.
> !se tiver interesse salva todo array Engloba.
> FIM.
>
> Porém para qualquer setor existir um ponto que seja sempre o que englobe
> mais pontos, creio que vá depender da nuvem, e.g.
>
> P1= (10,40)
> P2= ( 6,42)
> P3= (9,90)
> P4= (8,100)
> P5= (7,107)
> P6= (7,5; 108)
> Teremos para uma excursão de + 5 graus:
> Engloba (1) = 1;Engloba (2) = 0; Engloba (3) = 0; Engloba (4) = 0 ,
> Engloba (5) = 0 e Engloba (6) = 0
> P1 é o que engloba mais pontos.
> Para uma excursão de +10 graus:
> Engloba (1) = 1;Engloba (2) = 0; Engloba (3) = 1; Engloba (4) = 2 e
> Engloba (5) = 0 e Engloba (6) = 0
> P3 é o que engloba mais pontos.
>
> Espero ter compreendido o proposto e ajudado.
> Mas o que tem haver com soma de complexos, módulo mínimo???
>
> Saudações,
> PJMS
>
> Em sáb, 3 de nov de 2018 às 22:31, Bruno Visnadi <
> brunovisnadida...@gmail.com> escreveu:
>
>> Não entendi a pergunta - o que é uma excursão?
>>
>> Em sáb, 3 de nov de 2018 às 22:18, Jardiel Cunha 
>> escreveu:
>>
>>> Olá!
>>>
>>>
>>> Estou trabalhando em um projeto e um problema está me tirando o sono há
>>> algum tempo. Meu trabalho é na área de engenharia de microondas. A solução
>>> que eu encontrei até agora, acha soluções mas não satisfatórias... Não
>>> precisam fazer o problema, queria apenas uma luz em que caminho seguir.
>>>
>>>
>>> [Problema] Dados N pontos em um círculo, estou querendo achar um ponto
>>> dentro do círculo tal que: para qualquer valor de excursão em graus, eu
>>> garanta que não existe outro ponto que englobe mais pontos no círculo do
>>> que ele.
>>>
>>>
>>> Por exemplo: se eu der uma excursão de 80 graus... então eu quero um
>>> ponto tal que englobe o maior número possível desses N pontos estando ele
>>> no centro de um arco de 80 graus.
>>>
>>>
>>> Mais um exemplo: tenho 10 pontos. Queria um ponto x tal que ele será o
>>> centro de todos os arcos com o maior número possível de pontos.
>>>
>>>
>>> Primeira pergunta: isso é possível???
>>>
>>> Segunda pergunta: como calcular este ponto?
>>>
>>>
>>> Abs
>>>

[obm-l] Re: [obm-l] Re: [obm-l] [Problema] Achar o mínimo do valor absoluto de uma soma complexa

2018-11-05 Por tôpico Pedro José
Boa tarde!
Se entendi o que você quer, não entendi qual a relação com o mínimo de uma
soma complexa?
Para resolver o problema que você propõe, entendi:
(i) a excursão como a geração de um setor circular, a partir de um ponto
inicial, essa incursão tem dois sentidos, trigonométrico ou horário.
(ii) Englobar um ponto significa que o ponto pertença ao setor circular,
tem que saber se incluem-se os pontos de borda ou não, como não há
restrição vou considerar que sim.
(iii) Estou supondo que seu universo é plano.
Minha sugestão é defina o conjunto de pontos em coordenadas polares.
Defina a variável excursão, e dê a ela um sinal para definir o sentido,
Faça um programa.
Definir "arrays" dos pontos (caso não estejam em coordenadas polares, tem
que fazer uma sub-rotina para transformar as coordenadas em polares) Mod(I)
e Teta(I)
Defina um array de contagens
Definir uma rotina para contar o número de pontos. N
Defina uma sub-rotina Achapontos para determinar o índice Imax, cujo
Engloba(I) seja máximo.
! Comentário: Atentar que podem retornar mais do que um índice. Portanto
deve-se definir um array Pontonotável e uma variável de contagem Nmax.
E.g., se tiverem três pontos que englobem o número máximo de pontos, deve
retornar: Array contagem, com os valores dos índices dos pontos que têm o
máximo de Engloba, nas três primeiras posições e o Valor Nmax=3.
Aplique a sub-rotina de contagem no array de pontos e retorne com N.
Entre com o valor de excursão
Faça de I=1 a N
Tetamax= max (teta(I);teta(i)+excursão)
Tetamin=min(teta(I);teta(i)+excursão)
Engloba(I)=0
Faça de J=1 a N
Se (teta(J)<=tetamax e teta(J)>=tetamin e mod(J)<=mod(I).
! comentário: A relação engloba será reflexiva. Todo ponto engloba si
próprio.Caso não se aceite a borda é só tirar os iguais da lógica acima.
Engloba(I) = Engloba(I)+1
Fim SE;
Fim Faça
Fim faça
Aplica Sub-rotina acha pontos.
Salva temos o máximo de pontos englobados para uma excurção de [excursão]
para [Nmax] pontos englobando [engloba(Nmax)]
São eles:
Faça de I=1 até N
Pontonotável(I)
Fim faça.
!se tiver interesse salva todo array Engloba.
FIM.

Porém para qualquer setor existir um ponto que seja sempre o que englobe
mais pontos, creio que vá depender da nuvem, e.g.

P1= (10,40)
P2= ( 6,42)
P3= (9,90)
P4= (8,100)
P5= (7,107)
P6= (7,5; 108)
Teremos para uma excursão de + 5 graus:
Engloba (1) = 1;Engloba (2) = 0; Engloba (3) = 0; Engloba (4) = 0 , Engloba
(5) = 0 e Engloba (6) = 0
P1 é o que engloba mais pontos.
Para uma excursão de +10 graus:
Engloba (1) = 1;Engloba (2) = 0; Engloba (3) = 1; Engloba (4) = 2 e Engloba
(5) = 0 e Engloba (6) = 0
P3 é o que engloba mais pontos.

Espero ter compreendido o proposto e ajudado.
Mas o que tem haver com soma de complexos, módulo mínimo???

Saudações,
PJMS

Em sáb, 3 de nov de 2018 às 22:31, Bruno Visnadi <
brunovisnadida...@gmail.com> escreveu:

> Não entendi a pergunta - o que é uma excursão?
>
> Em sáb, 3 de nov de 2018 às 22:18, Jardiel Cunha 
> escreveu:
>
>> Olá!
>>
>>
>> Estou trabalhando em um projeto e um problema está me tirando o sono há
>> algum tempo. Meu trabalho é na área de engenharia de microondas. A solução
>> que eu encontrei até agora, acha soluções mas não satisfatórias... Não
>> precisam fazer o problema, queria apenas uma luz em que caminho seguir.
>>
>>
>> [Problema] Dados N pontos em um círculo, estou querendo achar um ponto
>> dentro do círculo tal que: para qualquer valor de excursão em graus, eu
>> garanta que não existe outro ponto que englobe mais pontos no círculo do
>> que ele.
>>
>>
>> Por exemplo: se eu der uma excursão de 80 graus... então eu quero um
>> ponto tal que englobe o maior número possível desses N pontos estando ele
>> no centro de um arco de 80 graus.
>>
>>
>> Mais um exemplo: tenho 10 pontos. Queria um ponto x tal que ele será o
>> centro de todos os arcos com o maior número possível de pontos.
>>
>>
>> Primeira pergunta: isso é possível???
>>
>> Segunda pergunta: como calcular este ponto?
>>
>>
>> Abs
>>
>>
>>
>> <https://www.avast.com/sig-email?utm_medium=email_source=link_campaign=sig-email_content=webmail_term=icon>
>>  Virus-free.
>> www.avast.com
>> <https://www.avast.com/sig-email?utm_medium=email_source=link_campaign=sig-email_content=webmail_term=link>
>> <#m_3082452599431825113_m_2503312673449891629_DAB4FAD8-2DD7-40BB-A1B8-4E2AA1F9FDF2>
>>
>> --
>> Esta mensagem foi verificada pelo sistema de antivírus e
>> acredita-se estar livre de perigo.
>>
>
> --
> Esta mensagem foi verificada pelo sistema de antivírus e
> acredita-se estar livre de perigo.

-- 
Esta mensagem foi verificada pelo sistema de antiv�rus e
 acredita-se estar livre de perigo.



[obm-l] Re: [obm-l] [Problema] Achar o mínimo do valor absoluto de uma soma complexa

2018-11-03 Por tôpico Bruno Visnadi
Não entendi a pergunta - o que é uma excursão?

Em sáb, 3 de nov de 2018 às 22:18, Jardiel Cunha 
escreveu:

> Olá!
>
>
> Estou trabalhando em um projeto e um problema está me tirando o sono há
> algum tempo. Meu trabalho é na área de engenharia de microondas. A solução
> que eu encontrei até agora, acha soluções mas não satisfatórias... Não
> precisam fazer o problema, queria apenas uma luz em que caminho seguir.
>
>
> [Problema] Dados N pontos em um círculo, estou querendo achar um ponto
> dentro do círculo tal que: para qualquer valor de excursão em graus, eu
> garanta que não existe outro ponto que englobe mais pontos no círculo do
> que ele.
>
>
> Por exemplo: se eu der uma excursão de 80 graus... então eu quero um ponto
> tal que englobe o maior número possível desses N pontos estando ele no
> centro de um arco de 80 graus.
>
>
> Mais um exemplo: tenho 10 pontos. Queria um ponto x tal que ele será o
> centro de todos os arcos com o maior número possível de pontos.
>
>
> Primeira pergunta: isso é possível???
>
> Segunda pergunta: como calcular este ponto?
>
>
> Abs
>
>
>
> 
>  Virus-free.
> www.avast.com
> 
> <#m_2503312673449891629_DAB4FAD8-2DD7-40BB-A1B8-4E2AA1F9FDF2>
>
> --
> Esta mensagem foi verificada pelo sistema de antivírus e
> acredita-se estar livre de perigo.
>

-- 
Esta mensagem foi verificada pelo sistema de antiv�rus e
 acredita-se estar livre de perigo.



[obm-l] [Problema] Achar o mínimo do valor absoluto de uma soma complexa

2018-11-03 Por tôpico Jardiel Cunha
Olá!


Estou trabalhando em um projeto e um problema está me tirando o sono há algum 
tempo. Meu trabalho é na área de engenharia de microondas. A solução que eu 
encontrei até agora, acha soluções mas não satisfatórias... Não precisam fazer 
o problema, queria apenas uma luz em que caminho seguir.


[Problema] Dados N pontos em um círculo, estou querendo achar um ponto dentro 
do círculo tal que: para qualquer valor de excursão em graus, eu garanta que 
não existe outro ponto que englobe mais pontos no círculo do que ele.


Por exemplo: se eu der uma excursão de 80 graus... então eu quero um ponto tal 
que englobe o maior número possível desses N pontos estando ele no centro de um 
arco de 80 graus.


Mais um exemplo: tenho 10 pontos. Queria um ponto x tal que ele será o centro 
de todos os arcos com o maior número possível de pontos.


Primeira pergunta: isso é possível???

Segunda pergunta: como calcular este ponto?


Abs

[https://ipmcdn.avast.com/images/icons/icon-envelope-tick-round-orange-animated-no-repeat-v1.gif]
Virus-free. 
www.avast.com

-- 
Esta mensagem foi verificada pelo sistema de antivírus e
 acredita-se estar livre de perigo.



Re: [obm-l] Soma de Produtos de Termos em PA

2018-08-30 Por tôpico Claudio Buffara
Suponhamos que a sequência (a1, a2, ..., an) com n >= 3 cumpra a condição
mas não seja PA.
Seja p o menor índice tal que:
(a1, ..., a(p-1)) é PA (digamos, de razão r) mas (a1, ..., a(p-1), ap) não
é PA.
Isso significa que ap - a(p-1) <> r  (&&&)

Como (a1, ..., a(p-1)) é PA, vale:
1/(a1*a2) + ... + 1/(a(p-2)*a(p-1)) = (p-2)/(a1*a(p-1))(*)

Além disso, por hipótese:
1/(a1*a2) + ... + 1/(a(p-1)*ap) = (p-1)/(a1*ap)(**)

Subtraindo (*) de (**), obtemos:
1/(a(p-1)*ap) = (1/a1)*((p-1)/ap - (p-2)/a(p-1))  ==>
a1/(a(p-1)*ap) = ((p-1)*a(p-1) - (p-2)*ap)/(ap*a(p-1)) ==>
a1 = a(p-1) + (p-2)*a(p-1) - (p-2)*ap ==>
a(p-1) = a1 + (p-2)*(ap - a(p-1))(***)

Mas, como (a1, ..., a(p-1)) é uma PA, vale a(p-1) = a1 + (p-2)*r  (),
onde r = razão da PA.
Comparando (***) e (), obtemos que ap - a(p-1) = r ==> contradição a
(&&&).

Logo, se uma sequência cumpre a condição, ela é PA.

[]s,
Claudio.


On Thu, Aug 30, 2018 at 3:21 PM Luís Lopes  wrote:

> Sauda,c~oes, oi Claudio,
>
> Seja S_{k-1} = (n-1)/(a1*an) = \frac{n-1}{a_1a_n}.
>
> Para provar a recíproca escrevi
>
> S_k = S_{k-1} + \frac{1}{a_n a_{n+1}} = \frac{n}{a_1a_{n+1}}
>
> e cheguei a
>
> n(a_{n+1} - a_n)=a_{n+1} - a_1 (*).
>
> Fazendo a) n=2 e b) n=3 em (*) tem-se
>
> a) a_3 + a_1 = 2a_2
>
> b) a_4 + a_2 = 2a_3
>
> Mas não consegui provar que a_{k+1} + a_{k-1} = 2a_k .
>
> Usando (*) ou de outra maneira, como provar a recíproca ?
>
> []s
> Luís
>
>
>
> --
> Esta mensagem foi verificada pelo sistema de antivírus e
> acredita-se estar livre de perigo.
>

-- 
Esta mensagem foi verificada pelo sistema de antiv�rus e
 acredita-se estar livre de perigo.



[obm-l] Soma de Produtos de Termos em PA

2018-08-30 Por tôpico Luís Lopes
Sauda,c~oes, oi Claudio,

Seja S_{k-1} = (n-1)/(a1*an) = \frac{n-1}{a_1a_n}.

Para provar a recíproca escrevi

S_k = S_{k-1} + \frac{1}{a_n a_{n+1}} = \frac{n}{a_1a_{n+1}}

e cheguei a

n(a_{n+1} - a_n)=a_{n+1} - a_1 (*).

Fazendo a) n=2 e b) n=3 em (*) tem-se

a) a_3 + a_1 = 2a_2

b) a_4 + a_2 = 2a_3

Mas não consegui provar que a_{k+1} + a_{k-1} = 2a_k .

Usando (*) ou de outra maneira, como provar a recíproca ?

[]s
Luís



-- 
Esta mensagem foi verificada pelo sistema de antivírus e
 acredita-se estar livre de perigo.



Re: [obm-l] Soma de Produtos de Termos em PA

2018-08-29 Por tôpico Kevin Felipe Kuhl Oliveira
Obrigado a todos!

Eu vou verificar se houve um erro de escrita. Provavelmente existe uma 
inconsistência mesmo. Legal essa propriedade da soma dos inversos dos produtos.

Um abraço

Kevin Kühl
On 29 Aug 2018 11:50 -0300, Claudio Buffara , wrote:
> A soma que você quer talvez seja a dos inversos dos produtos de termos 
> consecutivos.
> Numa PA a1, a2, ..., an, vale:
> 1/(a1*a2) + 1/(a2*a3) + ... + 1/(a(n-1)*an) = (n-1)/(a1*an).
>
> E vale também a recíproca: se uma sequência (a1, a2, a3, ...) é tal que para 
> todo n>=3 vale a igualdade acima, então a sequência é uma PA.
>
> []s,
> Claudio.
>
>
>
> > On Wed, Aug 29, 2018 at 9:28 AM Kevin Felipe Kuhl Oliveira 
> >  wrote:
> > > Bom dia, vocês já viram o seguinte problema?
> > >
> > > Sejam a1, a2, a3, ..., an termos consecutivos, não nulos, de uma PA, 
> > > nessa ordem. Mostre que
> > >
> > > (a1*a2) + (a2*a3) + ... + (a(n-1)*an) = (n-1)(a1*an)
> > >
> > > Na minha resposta aparece um termo com r^2 ao final, então não consigo 
> > > provar. Se alguém puder ajudar, agradeço.
> > >
> > > Um abraço
> > >
> > > Kevin Kühl
> > >
> > > --
> > > Esta mensagem foi verificada pelo sistema de antivírus e
> > > acredita-se estar livre de perigo.
>
> --
> Esta mensagem foi verificada pelo sistema de antiv�rus e
> acredita-se estar livre de perigo.

-- 
Esta mensagem foi verificada pelo sistema de antiv�rus e
 acredita-se estar livre de perigo.



Re: [obm-l] Soma de Produtos de Termos em PA

2018-08-29 Por tôpico Claudio Buffara
A soma que você quer talvez seja a dos inversos dos produtos de termos
consecutivos.
Numa PA a1, a2, ..., an, vale:
1/(a1*a2) + 1/(a2*a3) + ... + 1/(a(n-1)*an) = (n-1)/(a1*an).

E vale também a recíproca: se uma sequência (a1, a2, a3, ...) é tal que
para todo n>=3 vale a igualdade acima, então a sequência é uma PA.

[]s,
Claudio.



On Wed, Aug 29, 2018 at 9:28 AM Kevin Felipe Kuhl Oliveira <
kevin_k...@usp.br> wrote:

> Bom dia, vocês já viram o seguinte problema?
>
> Sejam a1, a2, a3, ..., an termos consecutivos, não nulos, de uma PA, nessa
> ordem. Mostre que
>
> (a1*a2) + (a2*a3) + ... + (a(n-1)*an) = (n-1)(a1*an)
>
> Na minha resposta aparece um termo com r^2 ao final, então não consigo
> provar. Se alguém puder ajudar, agradeço.
>
> Um abraço
>
> Kevin Kühl
>
> --
> Esta mensagem foi verificada pelo sistema de antivírus e
> acredita-se estar livre de perigo.
>

-- 
Esta mensagem foi verificada pelo sistema de antiv�rus e
 acredita-se estar livre de perigo.



Re: [obm-l] Soma de Produtos de Termos em PA

2018-08-29 Por tôpico Artur Steiner
Isso não é verdade. Se n  3,
a1 = 1, a2= 2, a3 = 3 então

a1 a2 + a2 a3 = 8
(n - 1) a1 an = 6

Não seria 2/(a1 a2) ... + 1/(a(n -1) an) = (n -1)/(a1 an)? Isso é verdade.

Artur Costa Steiner


Em qua, 29 de ago de 2018 09:28, Kevin Felipe Kuhl Oliveira <
kevin_k...@usp.br> escreveu:

> Bom dia, vocês já viram o seguinte problema?
>
> Sejam a1, a2, a3, ..., an termos consecutivos, não nulos, de uma PA, nessa
> ordem. Mostre que
>
> (a1*a2) + (a2*a3) + ... + (a(n-1)*an) = (n-1)(a1*an)
>
> Na minha resposta aparece um termo com r^2 ao final, então não consigo
> provar. Se alguém puder ajudar, agradeço.
>
> Um abraço
>
> Kevin Kühl
>
> --
> Esta mensagem foi verificada pelo sistema de antivírus e
> acredita-se estar livre de perigo.
>

-- 
Esta mensagem foi verificada pelo sistema de antiv�rus e
 acredita-se estar livre de perigo.



Re: [obm-l] Soma de Produtos de Termos em PA

2018-08-29 Por tôpico Claudio Buffara
Tá certo isso?
Tome a PA (1,2,3,4)  a1 = 1, an = n = 4
soma = 1*2 + 2*3 + 3*4 = 2 + 6 + 12 = 20.
Mas (n-1)*a1*an = 3*1*4 = 12.



On Wed, Aug 29, 2018 at 9:38 AM Claudio Buffara 
wrote:

> an = a1 + (n-1)r ==> r = (an - a1)/(n-1) ==> r^2 = (an - a1)^2/(n-1)^2.
> Use esta expressão pra r^2. Com alguma álgebra você deve chegar lá.
>
>
> On Wed, Aug 29, 2018 at 9:28 AM Kevin Felipe Kuhl Oliveira <
> kevin_k...@usp.br> wrote:
>
>> Bom dia, vocês já viram o seguinte problema?
>>
>> Sejam a1, a2, a3, ..., an termos consecutivos, não nulos, de uma PA,
>> nessa ordem. Mostre que
>>
>> (a1*a2) + (a2*a3) + ... + (a(n-1)*an) = (n-1)(a1*an)
>>
>> Na minha resposta aparece um termo com r^2 ao final, então não consigo
>> provar. Se alguém puder ajudar, agradeço.
>>
>> Um abraço
>>
>> Kevin Kühl
>>
>> --
>> Esta mensagem foi verificada pelo sistema de antivírus e
>> acredita-se estar livre de perigo.
>>
>

-- 
Esta mensagem foi verificada pelo sistema de antiv�rus e
 acredita-se estar livre de perigo.



Re: [obm-l] Soma de Produtos de Termos em PA

2018-08-29 Por tôpico Claudio Buffara
an = a1 + (n-1)r ==> r = (an - a1)/(n-1) ==> r^2 = (an - a1)^2/(n-1)^2.
Use esta expressão pra r^2. Com alguma álgebra você deve chegar lá.


On Wed, Aug 29, 2018 at 9:28 AM Kevin Felipe Kuhl Oliveira <
kevin_k...@usp.br> wrote:

> Bom dia, vocês já viram o seguinte problema?
>
> Sejam a1, a2, a3, ..., an termos consecutivos, não nulos, de uma PA, nessa
> ordem. Mostre que
>
> (a1*a2) + (a2*a3) + ... + (a(n-1)*an) = (n-1)(a1*an)
>
> Na minha resposta aparece um termo com r^2 ao final, então não consigo
> provar. Se alguém puder ajudar, agradeço.
>
> Um abraço
>
> Kevin Kühl
>
> --
> Esta mensagem foi verificada pelo sistema de antivírus e
> acredita-se estar livre de perigo.
>

-- 
Esta mensagem foi verificada pelo sistema de antiv�rus e
 acredita-se estar livre de perigo.



[obm-l] Soma de Produtos de Termos em PA

2018-08-29 Por tôpico Kevin Felipe Kuhl Oliveira
Bom dia, vocês já viram o seguinte problema?

Sejam a1, a2, a3, ..., an termos consecutivos, não nulos, de uma PA, nessa 
ordem. Mostre que

(a1*a2) + (a2*a3) + ... + (a(n-1)*an) = (n-1)(a1*an)

Na minha resposta aparece um termo com r^2 ao final, então não consigo provar. 
Se alguém puder ajudar, agradeço.

Um abraço

Kevin Kühl

-- 
Esta mensagem foi verificada pelo sistema de antiv�rus e
 acredita-se estar livre de perigo.



Re: [obm-l] Soma (k = 1, n) 1/P'(r_k) = 0

2018-04-16 Por tôpico gugu
   Dá para fazer com interpolação de Lagrange: o único polinômio de  
grau <=n-1 que vale 1 em r_i para 1<=i<=n (que obviamente é o  
polinômio constante igual a 1) é dado por
soma_(1<=i<=n)Produto_(j<>i)((x-r_j)/(r_i-r_j)). Aí é só olhar para o  
coeficiente de x^[n-1} (que é 0).

   Abraços,
 Gugu

Quoting Claudio Buffara <claudio.buff...@gmail.com>:


Resumo da ópera: ainda não temos uma demonstração elementar disso.

Mas não deixa de ser interessante tentar dar uma interpretação geométrica
da expressão para polinômios de grau baixo que tenham todas as raízes reais
e distintas.

Grau 2 é meio evidente:  as retas tangentes à parábola nas raízes têm
inclinações de mesma magnitude e sinais opostos.

Grau 3 é mais interessante...
De cara, dá margem ao problema: "Uma função polinomial de grau 3 tem raízes
(ou será que é melhor chamar de "zeros" - funções têm zeros, quem tem
raízes são equações...) reais e distintas: a, b e c. Se as retas tangentes
ao gráfico da função nos pontos (a,0) e (b,0) têm inclinações m e n,
respectivamente, determine a inclinação da reta tangente ao gráfico no
ponto (c,0).", cuja tentativa mais óbvia de solução vai descambar num monte
de contas (desnecessárias) envolvendo a, b e c, quando a resposta é
-mn/(m+n).

[]s,
Claudio.

2018-04-16 14:10 GMT-03:00 Douglas Oliveira de Lima <
profdouglaso.del...@gmail.com>:


Entao a questao é até que ponto ela é verdadeira , pois funciona para
casos elementares.

Douglas Oliveira

Em dom, 15 de abr de 2018 22:29, Bernardo Freitas Paulo da Costa <
bernardo...@gmail.com> escreveu:


2018-04-15 13:09 GMT-03:00 Douglas Oliveira de Lima
<profdouglaso.del...@gmail.com>:
> Usa o polinomio de Lagrange , nao é nada obvia mesmo.

Como usa Lagrange, a fórmula segue para k = 0, 1, ... n-1
(interpolando em n pontos, vamos até grau n-1).  E é, de fato, falso
para k = n, use P(x) = (x-1)(x+1).

Além disso, mesmo para k = n-1, a demonstração por complexa não se
aplica mais (o grau dá errado...), e o mesmo polinômio serve para
mostrar que a soma não dá mais zero.

Abraços,
--
Bernardo Freitas Paulo da Costa

--
Esta mensagem foi verificada pelo sistema de antivírus e
 acredita-se estar livre de perigo.


=
Instru�ões para entrar na lista, sair da lista e usar a lista em
http://www.mat.puc-rio.br/~obmlistas/obm-l.html
=



--
Esta mensagem foi verificada pelo sistema de antivírus e
acredita-se estar livre de perigo.



--
Esta mensagem foi verificada pelo sistema de antiv?rus e
 acredita-se estar livre de perigo.





--
Esta mensagem foi verificada pelo sistema de antiv�rus e
acredita-se estar livre de perigo.

=
Instru��es para entrar na lista, sair da lista e usar a lista em
http://www.mat.puc-rio.br/~obmlistas/obm-l.html
=


Re: [obm-l] Soma (k = 1, n) 1/P'(r_k) = 0

2018-04-16 Por tôpico Bernardo Freitas Paulo da Costa
2018-04-16 20:54 GMT-03:00 Claudio Buffara :
> Resumo da ópera: ainda não temos uma demonstração elementar disso.
>
> Mas não deixa de ser interessante tentar dar uma interpretação geométrica da
> expressão para polinômios de grau baixo que tenham todas as raízes reais e
> distintas.
>
> Grau 2 é meio evidente:  as retas tangentes à parábola nas raízes têm
> inclinações de mesma magnitude e sinais opostos.

A média harmônica das inclinações é zero, o que mesmo algébrico, não
deixa de ser interessante.  E talvez "baste" achar uma visão
geométrica da média harmônica neste contexto. Outra forma de dar a
mesma equação é que "a última inclinação" é (o oposto) da média
harmônica das outras.  Como interpretar isso, tenho menos ideia
ainda...


> Grau 3 é mais interessante...
> De cara, dá margem ao problema: "Uma função polinomial de grau 3 tem raízes
> (ou será que é melhor chamar de "zeros" - funções têm zeros, quem tem raízes
> são equações...) reais e distintas: a, b e c. Se as retas tangentes ao
> gráfico da função nos pontos (a,0) e (b,0) têm inclinações m e n,
> respectivamente, determine a inclinação da reta tangente ao gráfico no ponto
> (c,0).", cuja tentativa mais óbvia de solução vai descambar num monte de
> contas (desnecessárias) envolvendo a, b e c, quando a resposta é -mn/(m+n).

Também é verdade (graças ao Douglas e o Polinômio Interpolador de
Lagrange) que a/m + b/n - c(m+n)/mn = 0, ou seja, an + bm = c(m+n), o
que dá uma relação entre m e n em função das raízes.  Observe que isso
está "certo" (de novo do ponto de vista algébrico), pois o polinômio
tem 4 coeficientes, e além das 3 raízes, precisamos de um fator
multiplicativo, que pode ser dado de várias formas: tipicamente, é o
coeficiente de mais alto grau ou o valor em um ponto particular que
não seja zero, mas agora acabamos de ver que poderia até ser a
inclinação de uma tangente a uma raiz simples! (a, b, c, m determinam
tudo!)

Aliás, isso sugere que talvez haja uma demonstração puramente
algébrica para tudo isso, contando dimensões.  O número de
coeficientes do polinômio é (n+1).  Por outro lado, dados n valores
x_i (para as raízes), e n valores m_i (para as derivadas nestes
pontos), sabemos que há n-1 equações G(X_i, M_i) = 0 para "acertar a
dimensão".  Claro que as equações devem ser simétricas nos X_i e
M_i... mas isso ainda não basta para mostrar a forma especial \sum
X_i^k / M_i = 0...  Alguém tem uma ideia?  Por exemplo, já pode ser um
bom passo mostrar que as equações são homogêneas em X e M.

Abraços,
-- 
Bernardo Freitas Paulo da Costa

-- 
Esta mensagem foi verificada pelo sistema de antiv�rus e
 acredita-se estar livre de perigo.


=
Instru��es para entrar na lista, sair da lista e usar a lista em
http://www.mat.puc-rio.br/~obmlistas/obm-l.html
=


Re: [obm-l] Soma (k = 1, n) 1/P'(r_k) = 0

2018-04-16 Por tôpico Claudio Buffara
Resumo da ópera: ainda não temos uma demonstração elementar disso.

Mas não deixa de ser interessante tentar dar uma interpretação geométrica
da expressão para polinômios de grau baixo que tenham todas as raízes reais
e distintas.

Grau 2 é meio evidente:  as retas tangentes à parábola nas raízes têm
inclinações de mesma magnitude e sinais opostos.

Grau 3 é mais interessante...
De cara, dá margem ao problema: "Uma função polinomial de grau 3 tem raízes
(ou será que é melhor chamar de "zeros" - funções têm zeros, quem tem
raízes são equações...) reais e distintas: a, b e c. Se as retas tangentes
ao gráfico da função nos pontos (a,0) e (b,0) têm inclinações m e n,
respectivamente, determine a inclinação da reta tangente ao gráfico no
ponto (c,0).", cuja tentativa mais óbvia de solução vai descambar num monte
de contas (desnecessárias) envolvendo a, b e c, quando a resposta é
-mn/(m+n).

[]s,
Claudio.

2018-04-16 14:10 GMT-03:00 Douglas Oliveira de Lima <
profdouglaso.del...@gmail.com>:

> Entao a questao é até que ponto ela é verdadeira , pois funciona para
> casos elementares.
>
> Douglas Oliveira
>
> Em dom, 15 de abr de 2018 22:29, Bernardo Freitas Paulo da Costa <
> bernardo...@gmail.com> escreveu:
>
>> 2018-04-15 13:09 GMT-03:00 Douglas Oliveira de Lima
>> <profdouglaso.del...@gmail.com>:
>> > Usa o polinomio de Lagrange , nao é nada obvia mesmo.
>>
>> Como usa Lagrange, a fórmula segue para k = 0, 1, ... n-1
>> (interpolando em n pontos, vamos até grau n-1).  E é, de fato, falso
>> para k = n, use P(x) = (x-1)(x+1).
>>
>> Além disso, mesmo para k = n-1, a demonstração por complexa não se
>> aplica mais (o grau dá errado...), e o mesmo polinômio serve para
>> mostrar que a soma não dá mais zero.
>>
>> Abraços,
>> --
>> Bernardo Freitas Paulo da Costa
>>
>> --
>> Esta mensagem foi verificada pelo sistema de antivírus e
>>  acredita-se estar livre de perigo.
>>
>>
>> =
>> Instru�ões para entrar na lista, sair da lista e usar a lista em
>> http://www.mat.puc-rio.br/~obmlistas/obm-l.html
>> =
>>
>
> --
> Esta mensagem foi verificada pelo sistema de antivírus e
> acredita-se estar livre de perigo.
>

-- 
Esta mensagem foi verificada pelo sistema de antiv�rus e
 acredita-se estar livre de perigo.



Re: [obm-l] Soma (k = 1, n) 1/P'(r_k) = 0

2018-04-16 Por tôpico Artur Steiner
É verdadeira para todo polinômio de grau  n >= 2 que tenha n raízes
simples.

Artur Costa Steiner

Em Seg, 16 de abr de 2018 14:18, Douglas Oliveira de Lima <
profdouglaso.del...@gmail.com> escreveu:

> Entao a questao é até que ponto ela é verdadeira , pois funciona para
> casos elementares.
>
> Douglas Oliveira
>
> Em dom, 15 de abr de 2018 22:29, Bernardo Freitas Paulo da Costa <
> bernardo...@gmail.com> escreveu:
>
>> 2018-04-15 13:09 GMT-03:00 Douglas Oliveira de Lima
>> <profdouglaso.del...@gmail.com>:
>> > Usa o polinomio de Lagrange , nao é nada obvia mesmo.
>>
>> Como usa Lagrange, a fórmula segue para k = 0, 1, ... n-1
>> (interpolando em n pontos, vamos até grau n-1).  E é, de fato, falso
>> para k = n, use P(x) = (x-1)(x+1).
>>
>> Além disso, mesmo para k = n-1, a demonstração por complexa não se
>> aplica mais (o grau dá errado...), e o mesmo polinômio serve para
>> mostrar que a soma não dá mais zero.
>>
>> Abraços,
>> --
>> Bernardo Freitas Paulo da Costa
>>
>> --
>> Esta mensagem foi verificada pelo sistema de antivírus e
>>  acredita-se estar livre de perigo.
>>
>>
>> =
>> Instru�ões para entrar na lista, sair da lista e usar a lista em
>> http://www.mat.puc-rio.br/~obmlistas/obm-l.html
>> =
>>
>
> --
> Esta mensagem foi verificada pelo sistema de antivírus e
> acredita-se estar livre de perigo.

-- 
Esta mensagem foi verificada pelo sistema de antiv�rus e
 acredita-se estar livre de perigo.



Re: [obm-l] Soma (k = 1, n) 1/P'(r_k) = 0

2018-04-16 Por tôpico Douglas Oliveira de Lima
Entao a questao é até que ponto ela é verdadeira , pois funciona para casos
elementares.

Douglas Oliveira

Em dom, 15 de abr de 2018 22:29, Bernardo Freitas Paulo da Costa <
bernardo...@gmail.com> escreveu:

> 2018-04-15 13:09 GMT-03:00 Douglas Oliveira de Lima
> <profdouglaso.del...@gmail.com>:
> > Usa o polinomio de Lagrange , nao é nada obvia mesmo.
>
> Como usa Lagrange, a fórmula segue para k = 0, 1, ... n-1
> (interpolando em n pontos, vamos até grau n-1).  E é, de fato, falso
> para k = n, use P(x) = (x-1)(x+1).
>
> Além disso, mesmo para k = n-1, a demonstração por complexa não se
> aplica mais (o grau dá errado...), e o mesmo polinômio serve para
> mostrar que a soma não dá mais zero.
>
> Abraços,
> --
> Bernardo Freitas Paulo da Costa
>
> --
> Esta mensagem foi verificada pelo sistema de antivírus e
>  acredita-se estar livre de perigo.
>
>
> =
> Instru�ões para entrar na lista, sair da lista e usar a lista em
> http://www.mat.puc-rio.br/~obmlistas/obm-l.html
> =
>

-- 
Esta mensagem foi verificada pelo sistema de antiv�rus e
 acredita-se estar livre de perigo.



Re: Re: [obm-l] Soma (k = 1, n) 1/P'(r_k) = 0

2018-04-16 Por tôpico qedtexte

Sauda,c~oes,

Mandei a mensagem abaixo dessa na 6a.feira mas acho que no chegou.

Terminei a dita mensagem com a pergunta


Como concluir (seria possvel ?) a partir de (*)
que \sum_{k=1}^{n} \frac{1}{Q'(a_k)} = 0 ?

Na verdade o Gugu provara (*) para o caso real, ou seja, Q(x) e no
Q(z). Talvez seja possvel provar (*) para Q(z) com alguma 
adaptao.
Aqui vou provar a soma pedida para os polinmios com os coeficientes
reais.

Seja



1/Q(x) = \sum_{k=1}^{n} \frac{1}{Q'(a_k)}\frac{1}{x-a_k} (*)


obtida a partir de


\frac{P(x)}{Q(x)} = \sum_{k=1}^{n} \frac{P(a_k)}{Q'(a_k)}\frac{1}{x-a_k}

Igualando o denominador de todas as parcelas do somatrio (*) 
obtm-se
um polinmio no numerador cujo termo lder  x^{n-1} e seu 
coeficiente
vale \sum_{k=1}^{n} \frac{1}{Q'(a_k)}. Ento para n=2 esta soma tem 
que
ser igual a zero pois por (*) o polinmio obtido s possui o termo 
independente
1.

Abs,
Lus



 mensagem enviada na ltima 6a.feira
Sauda,c~oes,

H algum (bastante) tempo atrs o Gugu (seme permitem)
mandou para a lista a prova do seguinte resultado:


Sejam P(z) e Q(z) dois polinmios, de graus m e n, respectivamente,
e mn. Se todas as n razes a_k de Q(z) so razes 
simples,
ento a decomposio em fraes parciais de 
P(z)/Q(z)
pode ser expressa da seguinte maneira (\frac{A}{B}=A/B) :

\frac{P(z)}{Q(z)} = \sum_{k=1}^{n} \frac{P(a_k)}{Q'(a_k)}\frac{1}{z-a_k}
onde Q'(z)=\frac{d}{dz}Q(z).


Ento se P(z) = 1 para todo z,


1/Q(z) = \sum_{k=1}^{n} \frac{1}{Q'(a_k)}\frac{1}{z-a_k} (*)

Como concluir (seria possvel ?) a partir de (*)
que \sum_{k=1}^{n} \frac{1}{Q'(a_k)} = 0 ?

Abs,
Lus

--
Esta mensagem foi verificada pelo sistema de antiv�rus e
acredita-se estar livre de perigo.



Re: [obm-l] Soma (k = 1, n) 1/P'(r_k) = 0

2018-04-15 Por tôpico Bernardo Freitas Paulo da Costa
2018-04-15 13:09 GMT-03:00 Douglas Oliveira de Lima
<profdouglaso.del...@gmail.com>:
> Usa o polinomio de Lagrange , nao é nada obvia mesmo.

Como usa Lagrange, a fórmula segue para k = 0, 1, ... n-1
(interpolando em n pontos, vamos até grau n-1).  E é, de fato, falso
para k = n, use P(x) = (x-1)(x+1).

Além disso, mesmo para k = n-1, a demonstração por complexa não se
aplica mais (o grau dá errado...), e o mesmo polinômio serve para
mostrar que a soma não dá mais zero.

Abraços,
-- 
Bernardo Freitas Paulo da Costa

-- 
Esta mensagem foi verificada pelo sistema de antiv�rus e
 acredita-se estar livre de perigo.


=
Instru��es para entrar na lista, sair da lista e usar a lista em
http://www.mat.puc-rio.br/~obmlistas/obm-l.html
=


Re: [obm-l] Soma (k = 1, n) 1/P'(r_k) = 0

2018-04-15 Por tôpico Douglas Oliveira de Lima
Usa o polinomio de Lagrange , nao é nada obvia mesmo.

Douglas Oliveira.

Em sex, 13 de abr de 2018 13:41, Claudio Buffara <claudio.buff...@gmail.com>
escreveu:

> Essa identidade:
>  x^k=soma (i=1,...,n)(x_i)^k.P(x)/(x-x_i).P'(x_i)
> não me parece nada óbvia.
>
> []s,
> Claudio.
>
>
> 2018-04-13 5:56 GMT-03:00 Douglas Oliveira de Lima <
> profdouglaso.del...@gmail.com>:
>
>> Entao, sendo x^k=soma (i=1,...,n)(x_i)^k.P(x)/(x-x_i).P'(x_i) , é só
>> igualar os coeficientes de x^(n-1) e pronto, a identidade se torna ate mais
>> genérica
>>
>> Soma (i= 1, n) (x_i)^k/P'(x_i) = 0
>>
>> Obs: x_i sao raizes.
>>
>> Abraco
>>
>> Douglas Oliveira.
>>
>>
>>
>>
>> Em 8 de abr de 2018 20:50, "Artur Steiner" <artur.costa.stei...@gmail.com>
>> escreveu:
>>
>> Seja P um polinômio complexo, de grau n >= 2, que tenha n raízes simples
>> r_1, ... r_n. Mostre que Soma (k = 1, n) 1/P'(r_k) = 0.
>>
>> Para quem conhece um pouco de análise complexa, isto é corolário de um
>> resultado geral. Mas parece que pode ser provado sem análise complexa.
>>
>> Artur Costa Steiner
>>
>> --
>> Esta mensagem foi verificada pelo sistema de antivírus e
>> acredita-se estar livre de perigo.
>>
>>
>>
>> --
>> Esta mensagem foi verificada pelo sistema de antivírus e
>> acredita-se estar livre de perigo.
>>
>
>
> --
> Esta mensagem foi verificada pelo sistema de antivírus e
> acredita-se estar livre de perigo.

-- 
Esta mensagem foi verificada pelo sistema de antiv�rus e
 acredita-se estar livre de perigo.



Re: [obm-l] Soma (k = 1, n) 1/P'(r_k) = 0

2018-04-13 Por tôpico Claudio Buffara
Essa identidade:
 x^k=soma (i=1,...,n)(x_i)^k.P(x)/(x-x_i).P'(x_i)
não me parece nada óbvia.

[]s,
Claudio.


2018-04-13 5:56 GMT-03:00 Douglas Oliveira de Lima <
profdouglaso.del...@gmail.com>:

> Entao, sendo x^k=soma (i=1,...,n)(x_i)^k.P(x)/(x-x_i).P'(x_i) , é só
> igualar os coeficientes de x^(n-1) e pronto, a identidade se torna ate mais
> genérica
>
> Soma (i= 1, n) (x_i)^k/P'(x_i) = 0
>
> Obs: x_i sao raizes.
>
> Abraco
>
> Douglas Oliveira.
>
>
>
>
> Em 8 de abr de 2018 20:50, "Artur Steiner" <artur.costa.stei...@gmail.com>
> escreveu:
>
> Seja P um polinômio complexo, de grau n >= 2, que tenha n raízes simples
> r_1, ... r_n. Mostre que Soma (k = 1, n) 1/P'(r_k) = 0.
>
> Para quem conhece um pouco de análise complexa, isto é corolário de um
> resultado geral. Mas parece que pode ser provado sem análise complexa.
>
> Artur Costa Steiner
>
> --
> Esta mensagem foi verificada pelo sistema de antivírus e
> acredita-se estar livre de perigo.
>
>
>
> --
> Esta mensagem foi verificada pelo sistema de antivírus e
> acredita-se estar livre de perigo.
>

-- 
Esta mensagem foi verificada pelo sistema de antiv�rus e
 acredita-se estar livre de perigo.



Re: [obm-l] Soma (k = 1, n) 1/P'(r_k) = 0

2018-04-13 Por tôpico Artur Steiner
A prova por análise complexa baseia-se no fato de que, se P e Q  são
polinômios com grau(P) >= grau(Q) + 2 e f =Q/P, definida para P(z) <> 0,
então,

Soma (z em Z) Res(f, z) = 0  (*)

onde Z é o conjunto dos zeros de P e Res(f, z) é o resíduo de f em z, que é
pólo de f.

A prova disso baseia-se no teorema dos resíduos e no fato de que lim r -->
oo Integral (sobre Cr) f(z) dz = 0, sendo Cr a periferia do disco de centro
na origem e raio r.

Particularizando-se para o caso em que Q(z) = 1 para todo z,  f = 1/P e (*)
se reduz a

Soma (z em Z) Res(1/P, z) = 0 (**)

Se z é zero simples de P, então Res(1/P, z) = 1/P'(z), pois z é pólo
simples de 1/P (observe que, neste caso,  P'(z) <> 0). Supondo-se agora que
P tem grau n >= 2 e tem n zeros simples r_1,  r_n, (**) implica que

Soma (k = 1, n) 1/P'(r_k) = 0

conforme afirmado.

A chave da prova é que

lim r --> oo Integral (sobre Cr) f(z) dz = 0,

a qual não é difícil. Baseia-se nas propriedades da integral e nas dos
polinômios.

Artur Costa Steiner

Em 13 de abr de 2018 00:57, "Mórmon Santos" <mormonsan...@gmail.com>
escreveu:

Como é por análise complexa?

Em qui, 12 de abr de 2018 15:22, Artur Steiner <
artur.costa.stei...@gmail.com> escreveu:

> Para n >= 3, só consegui por análise complexa. Há uma prova que me parece
> muito bonita.
>
> Tentei também por frações parciais, mas caí num imbróglio.
>
> Artur Costa Steiner
>
> Em Qui, 12 de abr de 2018 14:42, Claudio Buffara <
> claudio.buff...@gmail.com> escreveu:
>
>> Olá! Alguém encontrou uma solução elementar par este?
>> Eu fiz pra n = 2 e n = 3 mas a generalização me parece muito complicada.
>>
>> []s,
>> Claudio.
>>
>>
>> 2018-04-08 20:42 GMT-03:00 Artur Steiner <artur.costa.stei...@gmail.com>:
>>
>>> Seja P um polinômio complexo, de grau n >= 2, que tenha n raízes simples
>>> r_1, ... r_n. Mostre que Soma (k = 1, n) 1/P'(r_k) = 0.
>>>
>>> Para quem conhece um pouco de análise complexa, isto é corolário de um
>>> resultado geral. Mas parece que pode ser provado sem análise complexa.
>>>
>>> Artur Costa Steiner
>>>
>>> --
>>> Esta mensagem foi verificada pelo sistema de antivírus e
>>> acredita-se estar livre de perigo.
>>
>>
>>
>> --
>> Esta mensagem foi verificada pelo sistema de antivírus e
>> acredita-se estar livre de perigo.
>
>
> --
> Esta mensagem foi verificada pelo sistema de antivírus e
> acredita-se estar livre de perigo.


-- 
Esta mensagem foi verificada pelo sistema de antivírus e
acredita-se estar livre de perigo.

-- 
Esta mensagem foi verificada pelo sistema de antiv�rus e
 acredita-se estar livre de perigo.



Re: [obm-l] Soma (k = 1, n) 1/P'(r_k) = 0

2018-04-13 Por tôpico Douglas Oliveira de Lima
Entao, sendo x^k=soma (i=1,...,n)(x_i)^k.P(x)/(x-x_i).P'(x_i) , é só
igualar os coeficientes de x^(n-1) e pronto, a identidade se torna ate mais
genérica

Soma (i= 1, n) (x_i)^k/P'(x_i) = 0

Obs: x_i sao raizes.

Abraco

Douglas Oliveira.




Em 8 de abr de 2018 20:50, "Artur Steiner" <artur.costa.stei...@gmail.com>
escreveu:

Seja P um polinômio complexo, de grau n >= 2, que tenha n raízes simples
r_1, ... r_n. Mostre que Soma (k = 1, n) 1/P'(r_k) = 0.

Para quem conhece um pouco de análise complexa, isto é corolário de um
resultado geral. Mas parece que pode ser provado sem análise complexa.

Artur Costa Steiner

-- 
Esta mensagem foi verificada pelo sistema de antivírus e
acredita-se estar livre de perigo.

-- 
Esta mensagem foi verificada pelo sistema de antiv�rus e
 acredita-se estar livre de perigo.



Re: [obm-l] Soma (k = 1, n) 1/P'(r_k) = 0

2018-04-12 Por tôpico Mórmon Santos
Como é por análise complexa?

Em qui, 12 de abr de 2018 15:22, Artur Steiner <
artur.costa.stei...@gmail.com> escreveu:

> Para n >= 3, só consegui por análise complexa. Há uma prova que me parece
> muito bonita.
>
> Tentei também por frações parciais, mas caí num imbróglio.
>
> Artur Costa Steiner
>
> Em Qui, 12 de abr de 2018 14:42, Claudio Buffara <
> claudio.buff...@gmail.com> escreveu:
>
>> Olá! Alguém encontrou uma solução elementar par este?
>> Eu fiz pra n = 2 e n = 3 mas a generalização me parece muito complicada.
>>
>> []s,
>> Claudio.
>>
>>
>> 2018-04-08 20:42 GMT-03:00 Artur Steiner <artur.costa.stei...@gmail.com>:
>>
>>> Seja P um polinômio complexo, de grau n >= 2, que tenha n raízes simples
>>> r_1, ... r_n. Mostre que Soma (k = 1, n) 1/P'(r_k) = 0.
>>>
>>> Para quem conhece um pouco de análise complexa, isto é corolário de um
>>> resultado geral. Mas parece que pode ser provado sem análise complexa.
>>>
>>> Artur Costa Steiner
>>>
>>> --
>>> Esta mensagem foi verificada pelo sistema de antivírus e
>>> acredita-se estar livre de perigo.
>>
>>
>>
>> --
>> Esta mensagem foi verificada pelo sistema de antivírus e
>> acredita-se estar livre de perigo.
>
>
> --
> Esta mensagem foi verificada pelo sistema de antivírus e
> acredita-se estar livre de perigo.

-- 
Esta mensagem foi verificada pelo sistema de antiv�rus e
 acredita-se estar livre de perigo.



Re: [obm-l] Soma (k = 1, n) 1/P'(r_k) = 0

2018-04-12 Por tôpico Artur Steiner
Para n >= 3, só consegui por análise complexa. Há uma prova que me parece
muito bonita.

Tentei também por frações parciais, mas caí num imbróglio.

Artur Costa Steiner

Em Qui, 12 de abr de 2018 14:42, Claudio Buffara <claudio.buff...@gmail.com>
escreveu:

> Olá! Alguém encontrou uma solução elementar par este?
> Eu fiz pra n = 2 e n = 3 mas a generalização me parece muito complicada.
>
> []s,
> Claudio.
>
>
> 2018-04-08 20:42 GMT-03:00 Artur Steiner <artur.costa.stei...@gmail.com>:
>
>> Seja P um polinômio complexo, de grau n >= 2, que tenha n raízes simples
>> r_1, ... r_n. Mostre que Soma (k = 1, n) 1/P'(r_k) = 0.
>>
>> Para quem conhece um pouco de análise complexa, isto é corolário de um
>> resultado geral. Mas parece que pode ser provado sem análise complexa.
>>
>> Artur Costa Steiner
>>
>> --
>> Esta mensagem foi verificada pelo sistema de antivírus e
>> acredita-se estar livre de perigo.
>
>
>
> --
> Esta mensagem foi verificada pelo sistema de antivírus e
> acredita-se estar livre de perigo.

-- 
Esta mensagem foi verificada pelo sistema de antiv�rus e
 acredita-se estar livre de perigo.



Re: [obm-l] Soma (k = 1, n) 1/P'(r_k) = 0

2018-04-12 Por tôpico Claudio Buffara
Olá! Alguém encontrou uma solução elementar par este?
Eu fiz pra n = 2 e n = 3 mas a generalização me parece muito complicada.

[]s,
Claudio.


2018-04-08 20:42 GMT-03:00 Artur Steiner <artur.costa.stei...@gmail.com>:

> Seja P um polinômio complexo, de grau n >= 2, que tenha n raízes simples
> r_1, ... r_n. Mostre que Soma (k = 1, n) 1/P'(r_k) = 0.
>
> Para quem conhece um pouco de análise complexa, isto é corolário de um
> resultado geral. Mas parece que pode ser provado sem análise complexa.
>
> Artur Costa Steiner
>
> --
> Esta mensagem foi verificada pelo sistema de antivírus e
> acredita-se estar livre de perigo.

-- 
Esta mensagem foi verificada pelo sistema de antiv�rus e
 acredita-se estar livre de perigo.



[obm-l] Soma (k = 1, n) 1/P'(r_k) = 0

2018-04-08 Por tôpico Artur Steiner
Seja P um polinômio complexo, de grau n >= 2, que tenha n raízes simples
r_1, ... r_n. Mostre que Soma (k = 1, n) 1/P'(r_k) = 0.

Para quem conhece um pouco de análise complexa, isto é corolário de um
resultado geral. Mas parece que pode ser provado sem análise complexa.

Artur Costa Steiner

-- 
Esta mensagem foi verificada pelo sistema de antiv�rus e
 acredita-se estar livre de perigo.



Re: [obm-l] soma de quadrados

2018-03-01 Por tôpico Claudio Buffara
3^2 + 4^2 = 5^2
5^2 + 12^2 = 13^2 ==> 3^2 + 4^2 + 12^2 = 13^2
13^2 + 84^2 = 85^2 ==> 3^2 + 4^2 + 12^2 + 84^2 = 85^2

Em geral, dado a^2 ímpar, você quer x tal que a^2 + x^2 = (x+1)^2 ==> x =
(a^2 -1)/2
a^2 = 85^2 ==> x = (85^2-1)/2 = 3612 ==>  3^2 + 4^2 + 12^2 + 84^2 + 3612^2
= 3613^2

Determinar a sequência que cresce mais devagar é outro problema...



2018-02-28 22:23 GMT-03:00 Bernardo Freitas Paulo da Costa <
bernardo...@gmail.com>:

> 2018-02-28 22:01 GMT-03:00 marcone augusto araújo borges
> <marconeborge...@hotmail.com>:
> > Seja a sequência
> >
> > 3^2 + 4^2 = 5^2
> > 3^2 + 4^2 + 12^2 = 13^2
> > 3^2 + 4^2 + 12^2 + 84^2 = 85^2
> >    .
> >.
> >.
> > A soma de n quadrados é um quadrado
> > Existe uma ´´lei de formação´´ ou uma recorrência para determinar
> > uma soma dessas para, digamos, n = 10 ou n = 30 ou n = 100, ...
>
> Vou dar (um) próximo termo.  Não é, necessariamente, o menor, nem o
> melhor, mas ele tem uma "lei de formação" fácil.
>
> 3^2 + 4^2 + 12^2 + 84^2 + 204^2 = 221^2
>
>
> A sequência que eu obtive tem crescimento "exponencial", ou seja, o
> n-ésimo termo é maior do que 2^n.  Seria interessante saber se existe
> uma sequência de crescimento polinomial...
>
> Abraços,
> --
> Bernardo
>
> --
> Esta mensagem foi verificada pelo sistema de antivírus e
>  acredita-se estar livre de perigo.
>
>
> =
> Instru�ões para entrar na lista, sair da lista e usar a lista em
> http://www.mat.puc-rio.br/~obmlistas/obm-l.html
> =
>

-- 
Esta mensagem foi verificada pelo sistema de antiv�rus e
 acredita-se estar livre de perigo.



Re: [obm-l] soma de quadrados

2018-02-28 Por tôpico Bernardo Freitas Paulo da Costa
2018-02-28 22:01 GMT-03:00 marcone augusto araújo borges
<marconeborge...@hotmail.com>:
> Seja a sequência
>
> 3^2 + 4^2 = 5^2
> 3^2 + 4^2 + 12^2 = 13^2
> 3^2 + 4^2 + 12^2 + 84^2 = 85^2
>.
>    .
>.
> A soma de n quadrados é um quadrado
> Existe uma ´´lei de formação´´ ou uma recorrência para determinar
> uma soma dessas para, digamos, n = 10 ou n = 30 ou n = 100, ...

Vou dar (um) próximo termo.  Não é, necessariamente, o menor, nem o
melhor, mas ele tem uma "lei de formação" fácil.

3^2 + 4^2 + 12^2 + 84^2 + 204^2 = 221^2


A sequência que eu obtive tem crescimento "exponencial", ou seja, o
n-ésimo termo é maior do que 2^n.  Seria interessante saber se existe
uma sequência de crescimento polinomial...

Abraços,
-- 
Bernardo

-- 
Esta mensagem foi verificada pelo sistema de antiv�rus e
 acredita-se estar livre de perigo.


=
Instru��es para entrar na lista, sair da lista e usar a lista em
http://www.mat.puc-rio.br/~obmlistas/obm-l.html
=


[obm-l] soma de quadrados

2018-02-28 Por tôpico marcone augusto araújo borges
Seja a sequência

3^2 + 4^2 = 5^2
3^2 + 4^2 + 12^2 = 13^2
3^2 + 4^2 + 12^2 + 84^2 = 85^2
   .
   .
   .
A soma de n quadrados é um quadrado
Existe uma ´´lei de formação´´ ou uma recorrência para determinar
uma soma dessas para, digamos, n = 10 ou n = 30 ou n = 100, ...

-- 
Esta mensagem foi verificada pelo sistema de antivírus e
 acredita-se estar livre de perigo.



Re: [obm-l] soma de tan^2

2017-09-16 Por tôpico Douglas Oliveira de Lima
Eu resolvi esse problema em 2014 aqui na lista olhe
https://www.mail-archive.com/obm-l@mat.puc-rio.br/msg52281.html

Abraços.

Em 16 de set de 2017 13:23, "Carlos Gomes"  escreveu:

Olá Luis...lembro desse problema ...ele foi publicado na Mathematical
excalibur ha alguns anos https://www.math.ust.hk/excalibur/

A resposta é C(90,2)= 4005, se não me falha a memória...usa relações de
Girard num "polinômio esperto"...vou tenter ver se lembro a solução...se
lembrar ponha aqui!

Abraço, Cgomes.

Em 16 de setembro de 2017 10:48, Luís Lopes 
escreveu:

> Sauda,c~oes,
>
>
> Bom dia.
>
>
> Me mandaram a seguinte questão:
>
>
> (1) Seja S = tan²(1º) + tan²(3º) + tan²(5º) + ... + tan²(89º), calcule o
> valor de S.
>
> Como resolver ? Obrigado.
>
>
> Abs,
>
> Luís
>
>
>
> --
> Esta mensagem foi verificada pelo sistema de antivírus e
> acredita-se estar livre de perigo.
>


-- 
Esta mensagem foi verificada pelo sistema de antivírus e
acredita-se estar livre de perigo.

-- 
Esta mensagem foi verificada pelo sistema de antiv�rus e
 acredita-se estar livre de perigo.



Re: [obm-l] soma de tan^2

2017-09-16 Por tôpico Carlos Gomes
Olá Luis...lembro desse problema ...ele foi publicado na Mathematical
excalibur ha alguns anos https://www.math.ust.hk/excalibur/

A resposta é C(90,2)= 4005, se não me falha a memória...usa relações de
Girard num "polinômio esperto"...vou tenter ver se lembro a solução...se
lembrar ponha aqui!

Abraço, Cgomes.

Em 16 de setembro de 2017 10:48, Luís Lopes 
escreveu:

> Sauda,c~oes,
>
>
> Bom dia.
>
>
> Me mandaram a seguinte questão:
>
>
> (1) Seja S = tan²(1º) + tan²(3º) + tan²(5º) + ... + tan²(89º), calcule o
> valor de S.
>
> Como resolver ? Obrigado.
>
>
> Abs,
>
> Luís
>
>
>
> --
> Esta mensagem foi verificada pelo sistema de antivírus e
> acredita-se estar livre de perigo.
>

-- 
Esta mensagem foi verificada pelo sistema de antiv�rus e
 acredita-se estar livre de perigo.



Re: [obm-l] soma de tan^2

2017-09-16 Por tôpico Israel Meireles Chrisostomo
acho que vc poderia trabalhar na expressão cis(nx) encontrar um polinômio e
usar a relação de girard

Em 16 de setembro de 2017 10:48, Luís Lopes 
escreveu:

> Sauda,c~oes,
>
>
> Bom dia.
>
>
> Me mandaram a seguinte questão:
>
>
> (1) Seja S = tan²(1º) + tan²(3º) + tan²(5º) + ... + tan²(89º), calcule o
> valor de S.
>
> Como resolver ? Obrigado.
>
>
> Abs,
>
> Luís
>
>
>
> --
> Esta mensagem foi verificada pelo sistema de antivírus e
> acredita-se estar livre de perigo.
>



-- 
Israel Meireles Chrisostomo

-- 
Esta mensagem foi verificada pelo sistema de antiv�rus e
 acredita-se estar livre de perigo.



[obm-l] soma de tan^2

2017-09-16 Por tôpico Luís Lopes
Sauda,c~oes,


Bom dia.


Me mandaram a seguinte questão:


(1) Seja S = tan²(1º) + tan²(3º) + tan²(5º) + ... + tan²(89º), calcule o valor 
de S.


Como resolver ? Obrigado.


Abs,

Luís


-- 
Esta mensagem foi verificada pelo sistema de antivírus e
 acredita-se estar livre de perigo.



[obm-l] Soma de duas frações irredutíveis (correção)

2016-11-25 Por tôpico Pedro Chaves
Caros Colegas,

Considerar a seguinte correção:  a, b, c e d são inteiros positivos.

-- 
Esta mensagem foi verificada pelo sistema de antivírus e
 acredita-se estar livre de perigo.



[obm-l] Re: [obm-l] Soma de duas frações irredutíveis

2016-11-25 Por tôpico Pedro José
Boa tarde!

a/b + c/d e (a,b)=1 e (c,d)=1

a/b + c/d = (ad+bc)/bd

Se a/b + c/d é inteiro ==> bd | (ad + bc) ==> b|d e d|b

b| d <=. |b| <= |d|
d | b ==> |d| <= |b|

Então temos que |b| = |d|.

Portanto, creio que deva ser inserida mais uma restrição no problema.

soma de duas frações irredutíveis," ..de denominadores *com módulos *
diferentes"

1/2 + 1/-2 = 0 e 2 <> -2

Saudações,
PJMS


Em 25 de novembro de 2016 10:01, Pedro Chaves <brped...@hotmail.com>
escreveu:

> Caros Colegas,
>
> Como provar que a soma de duas frações irredutíveis, de denominadores
> diferentes, nunca é um número inteiro?
>
> Abraços!
> Pedro Chaves
>
> --
> Esta mensagem foi verificada pelo sistema de antivírus e
> acredita-se estar livre de perigo.
>

-- 
Esta mensagem foi verificada pelo sistema de antiv�rus e
 acredita-se estar livre de perigo.



[obm-l] Soma de duas frações irredutíveis

2016-11-25 Por tôpico Pedro Chaves
Caros Colegas,

Como provar que a soma de duas frações irredutíveis, de denominadores 
diferentes, nunca é um número inteiro?

Abraços!
Pedro Chaves

-- 
Esta mensagem foi verificada pelo sistema de antivírus e
 acredita-se estar livre de perigo.



Re: [obm-l] soma binomial

2016-08-03 Por tôpico Anderson Torres
Uma ideia que pode funcionar, mas tem que ter alguma base para tentar:
séries formais.
Tente ver se existe uma série formal que descreva f(k), e basta
multiplicá-la por 1/(1-x).



Em 7 de julho de 2016 09:45, Luís Lopes <qed_te...@hotmail.com> escreveu:

> Sauda,c~oes, oi Anderson,
>
>
> > Deve ter alguma forma de passar isso para uma função hipergeométrica
>
> Deve ter. Tentei isso e só complicou.
>
>
> > e ver se de fato tem solução fácil.
>
> Ou melhor, uma solução esperta.
>
> Pelo que sei do problema, deve ter. Vem do
>
> Mathematical Reflections.
>
>
> > Dei uma trapaceada, mas parece que o Wolfram Alpha não reconhece.
>
> Ok. Nem pensei nisso. Mas acho que há programas capazes de
>
> fornecer a forma fechada.
>
>
> > Eu jogo diversos valores e isso tende a 1/3
>
> Verdade. Fica S_n = 1/3 - ??
>
>
> > - e o desejo de usar indução aumenta!
>
> Verdade. A solução apresentada usa indução.
>
> Mas acho nesse caso um pouco bastante artificial
>
> pois o - ?? - acima veio do nada. A indução em si é fácil.
>
>
> Na verdade comecei tentando S_n = \sum_{k=1}^n f(k)
>
> com
>
> f(k) = \frac{ k - 1 } { \binom{2k}{k} }
>
> pois achei que era mais fácil com este f(k) do que com este aqui:
>
> f(k) =  \frac{ 3k + 1 } { ( 2k + 1 ) \binom{2k}{k} } .
>
>
> Neste deu pra calcular F(k) tal que F(k+1) - F(k) = \Delta F(k) = f(k)
>
> e assim S_n = F(n+1) - F(1) = 1 - ?? .
>
>
> Pro f(k) = \frac{ k - 1 } { \binom{2k}{k} } deve ter uma manipulação
>
> binomial esperta pra obter o F(k) que não consigo ver.
>
>
> Abs,
>
> Luís
>
>
>
> --
> *De:* owner-ob...@mat.puc-rio.br <owner-ob...@mat.puc-rio.br> em nome de
> Anderson Torres <torres.anderson...@gmail.com>
> *Enviado:* quinta-feira, 7 de julho de 2016 02:17:43
> *Para:* obm-l@mat.puc-rio.br
> *Assunto:* Re: [obm-l] soma binomial
>
> Deve ter alguma forma de passar isso para uma função hipergeométrica e
> ver se de fato tem solução fácil.
>
> Dei uma trapaceada, mas parece que o Wolfram Alpha não reconhece. Eu
> jogo diversos valores e isso tende a 1/3 - e o desejo de usar indução
> aumenta!
>
> Em 6 de julho de 2016 15:19, Luís Lopes <qed_te...@hotmail.com> escreveu:
> > Sauda,c~oes,
> >
> >
> > Alguém saberia como resolver (sem computador e indução) ?
> >
> > S_n = \sum_{k=1}^n f(k)
> > com
> > f(k) = \frac{ k-1 } { \binom{2k}{k} }.
> >
> > Abs,
> > Luís
> >
> > --
> > Esta mensagem foi verificada pelo sistema de antivírus e
> > acredita-se estar livre de perigo.
>
> --
> Esta mensagem foi verificada pelo sistema de antiv�rus e
>  acredita-se estar livre de perigo.
>
>
> =
> Instru��es para entrar na lista, sair da lista e usar a lista em
> http://www.mat.puc-rio.br/~obmlistas/obm-l.html
> Lista obm-l - Departamento de Matemática - PUC-Rio
> <http://www.mat.puc-rio.br/~obmlistas/obm-l.html>
> www.mat.puc-rio.br
> Lista obm-l. Existem pelo menos dois arquivos da lista obm-l. Um deles
> fica bem aqui, em http://www.mat.puc-rio.br/~obmlistas/obm-l.arquivo.html
>
>
> =
>
> --
> Esta mensagem foi verificada pelo sistema de antivírus e
> acredita-se estar livre de perigo.
>

-- 
Esta mensagem foi verificada pelo sistema de antiv�rus e
 acredita-se estar livre de perigo.



Re: [obm-l] soma binomial

2016-07-07 Por tôpico Luís Lopes
Sauda,c~oes, oi Anderson,


> Deve ter alguma forma de passar isso para uma função hipergeométrica

Deve ter. Tentei isso e só complicou.


> e ver se de fato tem solução fácil.

Ou melhor, uma solução esperta.

Pelo que sei do problema, deve ter. Vem do

Mathematical Reflections.


> Dei uma trapaceada, mas parece que o Wolfram Alpha não reconhece.

Ok. Nem pensei nisso. Mas acho que há programas capazes de

fornecer a forma fechada.


> Eu jogo diversos valores e isso tende a 1/3

Verdade. Fica S_n = 1/3 - ??


> - e o desejo de usar indução aumenta!

Verdade. A solução apresentada usa indução.

Mas acho nesse caso um pouco bastante artificial

pois o - ?? - acima veio do nada. A indução em si é fácil.


Na verdade comecei tentando S_n = \sum_{k=1}^n f(k)

com

f(k) = \frac{ k - 1 } { \binom{2k}{k} }

pois achei que era mais fácil com este f(k) do que com este aqui:

f(k) =  \frac{ 3k + 1 } { ( 2k + 1 ) \binom{2k}{k} } .


Neste deu pra calcular F(k) tal que F(k+1) - F(k) = \Delta F(k) = f(k)

e assim S_n = F(n+1) - F(1) = 1 - ?? .


Pro f(k) = \frac{ k - 1 } { \binom{2k}{k} } deve ter uma manipulação

binomial esperta pra obter o F(k) que não consigo ver.


Abs,

Luís



De: owner-ob...@mat.puc-rio.br <owner-ob...@mat.puc-rio.br> em nome de Anderson 
Torres <torres.anderson...@gmail.com>
Enviado: quinta-feira, 7 de julho de 2016 02:17:43
Para: obm-l@mat.puc-rio.br
Assunto: Re: [obm-l] soma binomial

Deve ter alguma forma de passar isso para uma função hipergeométrica e
ver se de fato tem solução fácil.

Dei uma trapaceada, mas parece que o Wolfram Alpha não reconhece. Eu
jogo diversos valores e isso tende a 1/3 - e o desejo de usar indução
aumenta!

Em 6 de julho de 2016 15:19, Luís Lopes <qed_te...@hotmail.com> escreveu:
> Sauda,c~oes,
>
>
> Alguém saberia como resolver (sem computador e indução) ?
>
> S_n = \sum_{k=1}^n f(k)
> com
> f(k) = \frac{ k-1 } { \binom{2k}{k} }.
>
> Abs,
> Luís
>
> --
> Esta mensagem foi verificada pelo sistema de antivírus e
> acredita-se estar livre de perigo.

--
Esta mensagem foi verificada pelo sistema de antiv?rus e
 acredita-se estar livre de perigo.


=
Instru??es para entrar na lista, sair da lista e usar a lista em
http://www.mat.puc-rio.br/~obmlistas/obm-l.html
Lista obm-l - Departamento de Matemática - 
PUC-Rio<http://www.mat.puc-rio.br/~obmlistas/obm-l.html>
www.mat.puc-rio.br
Lista obm-l. Existem pelo menos dois arquivos da lista obm-l. Um deles fica bem 
aqui, em http://www.mat.puc-rio.br/~obmlistas/obm-l.arquivo.html



=

-- 
Esta mensagem foi verificada pelo sistema de antivírus e
 acredita-se estar livre de perigo.



Re: [obm-l] soma binomial

2016-07-06 Por tôpico Anderson Torres
Deve ter alguma forma de passar isso para uma função hipergeométrica e
ver se de fato tem solução fácil.

Dei uma trapaceada, mas parece que o Wolfram Alpha não reconhece. Eu
jogo diversos valores e isso tende a 1/3 - e o desejo de usar indução
aumenta!

Em 6 de julho de 2016 15:19, Luís Lopes  escreveu:
> Sauda,c~oes,
>
>
> Alguém saberia como resolver (sem computador e indução) ?
>
> S_n = \sum_{k=1}^n f(k)
> com
> f(k) = \frac{ k-1 } { \binom{2k}{k} }.
>
> Abs,
> Luís
>
> --
> Esta mensagem foi verificada pelo sistema de antivírus e
> acredita-se estar livre de perigo.

-- 
Esta mensagem foi verificada pelo sistema de antiv�rus e
 acredita-se estar livre de perigo.


=
Instru��es para entrar na lista, sair da lista e usar a lista em
http://www.mat.puc-rio.br/~obmlistas/obm-l.html
=


[obm-l] soma binomial

2016-07-06 Por tôpico Luís Lopes
Sauda,c~oes,

Alguém saberia como resolver (sem computador e indução) ?

S_n = \sum_{k=1}^n f(k)
com
f(k) = \frac{ k-1 } { \binom{2k}{k} }.

Abs,
Luís

-- 
Esta mensagem foi verificada pelo sistema de antivírus e
 acredita-se estar livre de perigo.



[obm-l] Re: [obm-l] Re: [obm-l] Re: [obm-l] Re: [obm-l] Soma dos números naturais

2016-03-03 Por tôpico Leonardo Maia
https://en.wikipedia.org/wiki/1_%2B_2_%2B_3_%2B_4_%2B_%E2%8B%AF

2016-03-03 14:24 GMT-03:00 Sávio Ribas <savio.ri...@gmail.com>:

> Vi uma palestra sobre isso (entre outras coisas) na última semana. O fato
> é que a Zeta(-1) = -1/12, onde Zeta(s) é a continuação analítica de 1 +
> 1/2^s + 1/3^s + ... para o plano complexo. Essa série só converge se a
> parte real de s é maior que 1, então não faz sentido fazer s = -1 e obter 1
> + 2 + 3 + ... = -1/12. Porém é verdade que alguns experimentos da física
> isso "se torna verdade", no sentido de que algum valor teórico era para dar
> algo que se comporta como 1 + 2 + 3 + ... mas na prática é medido -1/12. O
> exemplo dado na palestra foi de sobreposições de ondas com comprimentos 1,
> 1/2, 1/3, ..., mas a partir daqui não sei mais nada (física, né...).
>
> Em 3 de março de 2016 14:13, Pedro Henrique <pedrohm...@gmail.com>
> escreveu:
>
>> Também achei isso mas existem diversos vídeos no YouTube q provam tal
>> afirmação.
>> Em 3 de mar de 2016 2:11 PM, "Alexandre Antunes" <
>> prof.alexandreantu...@gmail.com> escreveu:
>>
>>>
>>> Essa afirmação parece estranha, pois a intuição parece indicar que essa
>>> soma tende para o infinito!
>>> Em 03/03/2016 13:54, "Pedro Henrique" <pedrohm...@gmail.com> escreveu:
>>>
>>>> Boa tarde!
>>>>
>>>> A um bom tempo atrás vi diversas explicações e também aplicações
>>>> práticas na física sobre a soma dos números naturais ser igual a -1/12 mas
>>>> não dei muita importância até que um aluno veio me questionar hj sobre a
>>>> veracidade deste problema, portanto gostaria de saber de vcs se essa
>>>> resposta está realmente correta e se n estiver quais contra-argumentos
>>>> podem ser usados.
>>>>
>>>> Obrigado!
>>>>
>>>> --
>>>> Esta mensagem foi verificada pelo sistema de antivírus e
>>>> acredita-se estar livre de perigo.
>>>
>>>
>>> --
>>> Esta mensagem foi verificada pelo sistema de antivírus e
>>> acredita-se estar livre de perigo.
>>
>>
>> --
>> Esta mensagem foi verificada pelo sistema de antivírus e
>> acredita-se estar livre de perigo.
>>
>
>
> --
> Esta mensagem foi verificada pelo sistema de antivírus e
> acredita-se estar livre de perigo.
>

-- 
Esta mensagem foi verificada pelo sistema de antiv�rus e
 acredita-se estar livre de perigo.



[obm-l] Re: [obm-l] Re: [obm-l] Re: [obm-l] Soma dos números naturais

2016-03-03 Por tôpico Sávio Ribas
Vi uma palestra sobre isso (entre outras coisas) na última semana. O fato é
que a Zeta(-1) = -1/12, onde Zeta(s) é a continuação analítica de 1 + 1/2^s
+ 1/3^s + ... para o plano complexo. Essa série só converge se a parte real
de s é maior que 1, então não faz sentido fazer s = -1 e obter 1 + 2 + 3 +
... = -1/12. Porém é verdade que alguns experimentos da física isso "se
torna verdade", no sentido de que algum valor teórico era para dar algo que
se comporta como 1 + 2 + 3 + ... mas na prática é medido -1/12. O exemplo
dado na palestra foi de sobreposições de ondas com comprimentos 1, 1/2,
1/3, ..., mas a partir daqui não sei mais nada (física, né...).

Em 3 de março de 2016 14:13, Pedro Henrique <pedrohm...@gmail.com> escreveu:

> Também achei isso mas existem diversos vídeos no YouTube q provam tal
> afirmação.
> Em 3 de mar de 2016 2:11 PM, "Alexandre Antunes" <
> prof.alexandreantu...@gmail.com> escreveu:
>
>>
>> Essa afirmação parece estranha, pois a intuição parece indicar que essa
>> soma tende para o infinito!
>> Em 03/03/2016 13:54, "Pedro Henrique" <pedrohm...@gmail.com> escreveu:
>>
>>> Boa tarde!
>>>
>>> A um bom tempo atrás vi diversas explicações e também aplicações
>>> práticas na física sobre a soma dos números naturais ser igual a -1/12 mas
>>> não dei muita importância até que um aluno veio me questionar hj sobre a
>>> veracidade deste problema, portanto gostaria de saber de vcs se essa
>>> resposta está realmente correta e se n estiver quais contra-argumentos
>>> podem ser usados.
>>>
>>> Obrigado!
>>>
>>> --
>>> Esta mensagem foi verificada pelo sistema de antivírus e
>>> acredita-se estar livre de perigo.
>>
>>
>> --
>> Esta mensagem foi verificada pelo sistema de antivírus e
>> acredita-se estar livre de perigo.
>
>
> --
> Esta mensagem foi verificada pelo sistema de antivírus e
> acredita-se estar livre de perigo.
>

-- 
Esta mensagem foi verificada pelo sistema de antiv�rus e
 acredita-se estar livre de perigo.



[obm-l] Re: [obm-l] Re: [obm-l] Soma dos números naturais

2016-03-03 Por tôpico Pedro Henrique
Também achei isso mas existem diversos vídeos no YouTube q provam tal
afirmação.
Em 3 de mar de 2016 2:11 PM, "Alexandre Antunes" <
prof.alexandreantu...@gmail.com> escreveu:

>
> Essa afirmação parece estranha, pois a intuição parece indicar que essa
> soma tende para o infinito!
> Em 03/03/2016 13:54, "Pedro Henrique" <pedrohm...@gmail.com> escreveu:
>
>> Boa tarde!
>>
>> A um bom tempo atrás vi diversas explicações e também aplicações práticas
>> na física sobre a soma dos números naturais ser igual a -1/12 mas não dei
>> muita importância até que um aluno veio me questionar hj sobre a veracidade
>> deste problema, portanto gostaria de saber de vcs se essa resposta está
>> realmente correta e se n estiver quais contra-argumentos podem ser usados.
>>
>> Obrigado!
>>
>> --
>> Esta mensagem foi verificada pelo sistema de antivírus e
>> acredita-se estar livre de perigo.
>
>
> --
> Esta mensagem foi verificada pelo sistema de antivírus e
> acredita-se estar livre de perigo.

-- 
Esta mensagem foi verificada pelo sistema de antiv�rus e
 acredita-se estar livre de perigo.



[obm-l] Re: [obm-l] Soma dos números naturais

2016-03-03 Por tôpico Alexandre Antunes
Essa afirmação parece estranha, pois a intuição parece indicar que essa
soma tende para o infinito!
Em 03/03/2016 13:54, "Pedro Henrique" <pedrohm...@gmail.com> escreveu:

> Boa tarde!
>
> A um bom tempo atrás vi diversas explicações e também aplicações práticas
> na física sobre a soma dos números naturais ser igual a -1/12 mas não dei
> muita importância até que um aluno veio me questionar hj sobre a veracidade
> deste problema, portanto gostaria de saber de vcs se essa resposta está
> realmente correta e se n estiver quais contra-argumentos podem ser usados.
>
> Obrigado!
>
> --
> Esta mensagem foi verificada pelo sistema de antivírus e
> acredita-se estar livre de perigo.

-- 
Esta mensagem foi verificada pelo sistema de antiv�rus e
 acredita-se estar livre de perigo.



  1   2   3   4   5   6   7   >